UK*TE Dec 21 Flashcards

1
Q

“Gagey’s sign” is elicited by stabilising the scapula with one hand, and abducting the arm with the other. Achieving abduction beyond 105 degrees is regarded as abnormal, and is a sign of:

a. Inferior laxity
b. Posterior laxity
c. Superior laxity
d. Anterior laxity
e. A large bony lesion

A

Answer: A

Gagey sign - hyperabduction test for assessment of the IGHL - range passive abduction >105 deg with 90 in the contralateral shoulder in 85% of patients with instability. An RPA of more than 105° is associated with lengthening and laxity of the inferior glenohumeral ligament.
Passive abduction occurs within the glenohumeral joint only, is controlled by the inferior glenohumeral ligament

Traumatic Anterior Shoulder Instability, also referred to as TUBS (Traumatic Unilateral dislocations with a Bankart lesion requiring Surgery), are traumatic shoulder injuries that generally occur as a result of an anterior force to the shoulder while its abducted and externally rotated and may lead to recurrent anterior shoulder instability.
Diagnosis is made clinically with the presence of positive anterior instability provocative tests and confirmed with MRI studies that may reveal labral and/or bony injuries of the glenoid and proximal humerus (Hill-Sachs lesion).
Treatment may be nonoperative or operative depending on the chronicity of symptoms, the presence of risk factors for recurrence, and the severity of labral and/or glenoid defects. In high-risk populations, surgery is often offered after a single dislocation event.

bankart lesion
is an avulsion of the anterior labrum and anterior band of the IGHL from the anterior inferior glenoid.
is present in 80-90% of patients with TUBS

bony bankart lesion
is a fracture of the anterior inferior glenoid
present in up to 49% of patients with recurrent dislocations
higher risk of failure of arthroscopic treatment if not addressed
defect >20-25% is considered “critical bone loss” and is biomechanically highly unstable
stability cannot be restored with soft tissue stabilization alone (unacceptable >2/3 failure rate)
requires bony procedure to restore bone loss (Latarjet-Bristow, other sources of autograft or allograft)

Humeral avulsion of the glenohumeral ligament (HAGL) occurs in patients slightly older than those with Bankart lesions
associated with a higher recurrence rate if not recognized and repaired
an indication for possible open surgical repair

glenoid labral articular defect (GLAD) is a sheared off portion of articular cartilage along with the labrum
presence is a risk factor for failure following arthroscopic stabilization procedures

anterior labral periosteal sleeve avulsion (ALPSA)
can cause torn labrum to heal medially along the medial glenoid neck
associated with higher failure rates following arthroscopic repair
common finding in patients with recurrent instability managed nonoperatively
97% of patients with recurrent instability have either a Bankart or ALPSA lesion

Hill-Sachs defect
is a chondral impaction injury in the posterosuperior humeral head secondary to contact with the glenoid rim.
is present in 80%-100% of traumatic dislocations and 25% of traumatic subluxations

Static restraints: bony anatomy, capsule, glenohumeral ligaments, labrum (labrum contributes 50% of additional glenoid depth)

Dynamic restraints: rotator cuff muscles & long head of biceps tendon

Anterior static shoulder stability is provided by:
1. Anterior band of IGHL (main restraint)
provides static restraint with arm in 90° of abduction and external rotation
2. MGHL - provides static restraint with arm in 45° of abduction and external rotation
3. SGHL - provides static restraint with arm at the side

How well did you know this?
1
Not at all
2
3
4
5
Perfectly
2
Q
  1. Which of the following tests is positive in thoracic outlet syndrome?

a. O’Brien’s test
b. Speed’s test
c. Hawkin’s sing
d. Hornblower’s sign
e. Adson’s test

A

Answer: E
Thoracic outlet syndrome is a neurovascular disorder resulting from compression of the brachial plexus and/or subclavian vessels in the interval between the neck and axilla.
Diagnosis can be suspected clinically with specific provocative tests and supplemented with radiographs or vascular studies. showing anatomic causes of compression.
Treatment may be nonoperative or include surgical decompression or a vascular procedure depending on the specific etiology.

neurogenic is most common (95%)
vascular may be venous (4%) or arterial (< 1%)
- hypertrophy of anterior scalene
- scalenus minimus: accessory muscle found in 30-50% of patients with TOS. Originates from cervical transverse process and inserts onto 1st rib between the subclavian artery and T1 root
-fibromuscular bands: increase stiffness and decrease compliance of the thoracic outlet
- costoclavicular ligament: abnormal insertion implicated in Paget-Schroetter syndrome (intermittent obstruction of subclavian vein in costoclavicular space –> upper extremity DVT)
- soft tissue tumors: Pancoast tumor, neuroblastoma, schwannoma brachial plexus
- abnormal pec minor

Osseous:
- cervical rib (arises from C7 vertebra)
- prominent C7 transverse process
- abnormal clavicle or 1st rib
- ACJ or SCJ injury or dislocation
- Osseous tumours - bone mets, osteoid osteoma

  • Chronic overuse - repetitive lifting - weight lifters, rowers, swimmers

Thoracic outlet is composed of 3 distinct spaces:
- interscalene triangle: brachial plexus trunks, subclavian artery
- costoclavicular space: brachial plexus divisions, subclavian artery and vein
- retropectoralis minor space: brachial plexus cords, axillary artery and vein

Adson Test - evaluates for compression at the interscalene triangle - patient seated, shoulder slightly abducted, externally rotated, elbow extended, forearm supinated - palpate radial pulse, then get patient to maximally extend and laterally rotate the neck towards side being tested, then inhales and holds breath + result is reduction amplitude /loss of radial pulse or reproduction pain/paraesthesia.

a. O’Brien’s test = active compression test +for SLAP is pain deep in GHJ while forearm is pronated, but not when supinated. Forward flex arm to 90deg, keep elbow fully extended, adduct arm to 10-15deg across body, pronate forearm (thumb pointing down) then examiner applies downward force and patient resists. Patient then supinates forearm and examiner applies pressure while patient resists.

b. Speed’s test - biceps injury - +is pain elicited in bicipital groove - forward elevate shoulder against resistance with elbow extended and forearm supinated.

c. Hawkin’s sign - impingement - flex shoulder and elbow to 90 and forcibly IR driving the greater tuberosity farther under the CA ligament.

d. Hornblower’s sign - terms minor - bring shoulder to 90deg abduction and ER, ask patient to hold this position, +if falls into IR

How well did you know this?
1
Not at all
2
3
4
5
Perfectly
3
Q
  1. In a patient with vascular concerns, which of the following is NOT a reason for performing a trans-radial amputation at the junction of the middle and distal third, rather than a more distal site?

A. The distal subcutaneous tissue is scant and less ideal for fashioning a stump
B. The forearm skin is thinner distally
C. Underlying soft tissue structures are relatively more avascular distally
D. Prosthesis fitting is easier
E. High incidence of complications with distal site

A

Answer: D

Easier prosthesis fitting is not a reason for a more proximal amputation

How well did you know this?
1
Not at all
2
3
4
5
Perfectly
4
Q
  1. Shoulder provocation test where pain is elicited when the patients shoulder is flexed to 90deg, adducted 15deg, internally rotated and a downward force applied is most suggestive of:
    a. Superior labral anterior to posterior (SLAP) tear
    b. Inferior glenohumeral ligament tear
    c. Middle glenohumeral ligament tear
    d. Supraspinatus tear
    e. Glenohumeral arthritis
A

Answer: A

• Active Compression test (“O’Brien’s Test”) - positive for SLAP tear when there is pain is “deep” in the glenohumeral joint while the forearm is pronated but not when the forearm is supinated. Technique.
o patient forward flexes the affected arm to 90 degrees while keeping the elbow fully extended. The arm is then adducted 10-15 degrees across the body. The patient then pronates the forearm so the thumb is pointing down. The examiner applies downward force to the wrist while the arm is in this position while the patient resists. The patient then supinates the forearm so the palm is up and the examiner once again applies force to the wrist while the patient resists.

Answer: Superior labral anterior to posterior (SLAP) tear – O’Brien’s test.

Impingement:
- Neer Impingement: indicative of impingement of rotator cuff tendon/bursa against the coracoacromial arch. Use one hand to prevent motion of the scapula; raise the arm of the patient with the other hand in forced elevation (somewhere between flexion and abduction) - pain is elicited (positive test) as the greater tuberosity impinges against the acromion (between 70-110°)
- Hawkins Test: flex shoulder to 90°, flex elbow to 90°, and forcibly internally rotate driving the greater tuberosity farther under the CA ligament.

Subscapularis Tests:
- Internal rotation lag sign = most sensitive and specific test for subscap pathology. stand behind patient, flex elbow to 90°, hold shoulder at 20° elevation and 20° extension. Internally rotate shoulder to near maximum holding the wrist by passively lifting the dorsum of the hand away from the lumbar spine – then supporting the elbow, tell patient to maintain position and release the wrist while looking for a lag.
- Lift off test, Belly press, Increased passive ER

Supraspinatus Tests:
• Jobe’s Test: weakness and/or impingement. Abduct arm to 90°, angle forward 30° (bringing it into the scapular plane), and internally rotate (thumb pointing to floor). Then press down on arm while patient attempts to maintain position testing for weakness or pain.
• Drop Sign: for function/integrity of supraspinatus. Passively elevate arm in scapular plan to 90°. Then ask the patient to slowly lower the arm. The test is positive when weakness or pain causes them to drop the arm to their side.
o most specific test for full thickness rotator cuff tear (specificity 98%)

infraspinatus Tests:
- External rotation lag sign

Teres Minor:
- Hornblower’s sign: bring the shoulder to 90 degrees of abduction, 90 degrees of external rotation and ask the patient to hold this position. Positive if the arm falls into internal rotation

How well did you know this?
1
Not at all
2
3
4
5
Perfectly
5
Q
  1. Which of the following is not a contraindication to total shoulder arthroplasty?

a. Non functioning deltoid
b. Complete tear of subscapularis and supraspinatus
c. Charcot arthropathy
d. Active infection
e. B2 glenoid

A

Answer: E

Replacement of humeral head and glenoid resurfacing
cemented all-polyethylene glenoid resurfacing is standard of care
Total shoulder arthroplasty unique from THA and TKA in that:
- greater range of motion in the shoulder
- success depends on proper functioning of the soft tissues
- glenoid is less constrained
- leads to greater sheer stresses and is more susceptible to mechanical loosening

Factors required for success of TSA
- Rotator cuff intact and functional: if rotator cuff is deficient and proximal migration of humerus is seen on x-rays (rotator cuff arthropathy) then glenoid resurfacing is contraindicated. If there is an irreparable rotator cuff deficiency then proceed with hemiarthroplasty or a reverse ball prosthesis
an isolated supraspinatus tear without retraction can proceed with TSA (incidence of full thickness rotator cuff tears in patients getting a TSA is 5% to 10%). If positive impingement signs on exam, order a pre-operative MRI

  • Glenoid bone stock and version: if glenoid is eroded down to coracoid process then glenoid resurfacing is contraindicated

Outcomes
pain relief most predictive benefit (more predictable than hemiarthroplasty)
reliable range of motion
good survival at 10 years (93%)
good longevity with cemented and press-fit humeral components
worse results for post-capsulorrhaphy arthropathy

How well did you know this?
1
Not at all
2
3
4
5
Perfectly
6
Q
  1. You are called to see a 76 year old diabetic male patient admitted yesterday with sepsis and a 4 week history of low back pain. He denies bladder or bowel symptoms and is ambulating with mild discomfort around the ward. Blood cultures have grown gram positive cocci and he has been started on empirical antibiotics. The medical team arrange whole spine MRI which shows discitis at the L4/5 level and an associated epidural abscess compressing the thecal sac including the cauda equina. On examination he is pyrexial, has absent sensation to the tips of his toes but normal perianal sensation. The most appropriate management is?

a. Biopsy of disc material with samples to microbiology
b. Percutaneous stabilisation
c. Debridement of epidural abscess via a posterior approach
d. Continued IV antibiotics with observations
e. Debridement of infection disc via anterior approach and stabilisation

A

Answer: D

How well did you know this?
1
Not at all
2
3
4
5
Perfectly
7
Q
  1. Which of the following changes are not seen in adult Hallux Valgus?

a. Plantar migration of abductor hallucis
b. Lateral migration of extensor hallucis longus
c. Attenuation of medial capsule
d. Supination of great toe
e. Increased 1st-2nd toe intermetatarsal angle

A

Answer: D

Hallux Valgus, commonly referred to as a bunion, is a complex valgus deformity of the first ray that can cause medial big toe pain and difficulty with shoe wear.

Risk factors

intrinsic:
genetic predisposition (70% of pts with hallux valgus have family history)
increased distal metaphyseal articular angle (DMAA)
ligamentous laxity (1st tarso-metatarsal joint instability)
convex metatarsal head
2nd toe deformity/amputation
pes planus
rheumatoid arthritis
cerebral palsy

extrinsic: shoes with high heel and narrow toe box

Pathoanatamy
- Valgus deviation of phalanx promotes varus position of metatarsal
- the metatarsal head displaces medially, leaving the sesamoid complex laterally translated relative to the metatarsal head
- Sesamoids remain within the respective head of the flexor hallucis brevis tendon and are attached to the base of the proximal phalanx via the sesamoido-phalangeal ligament
- this lateral displacement can lead to transfer metatarsalgia due to shift in weight bearing
- medial MTP joint capsule becomes stretched and attenuated while the lateral capsule becomes contracted
- adductor tendon becomes deforming force (inserts on fibular sesamoid and lateral aspect of proximal phalanx)
- lateral deviation of EHL further contributes to deformity
- plantar and lateral migration of the abductor hallucis causes muscle to plantar flex and pronate phalanx
- windlass mechanism becomes less effective - leads to transfer metatarsalgia

Normal IMA <9 deg

Operative Mx:

distal osteotomy: indicated in mild disease (IMA < 13)

proximal or combined osteotomy: indicated in more moderate disease (IMA > 13)

1st TMT arthrodesis: arthritis at TMT joint or instability

fusion procedures: indicated in severe deformity/spasticity/arthritis

MTP resection arthroplasty: only indicated in elderly patients with low functional demands

How well did you know this?
1
Not at all
2
3
4
5
Perfectly
8
Q
  1. During the approach for an anterior L5/S1 interbody fusion, you are mobilising the vessels and encounter significant bleeding. Which vessel has most likely been injured?
    a. Ilio-lumbar vein
    b. Left common iliac artery
    c. Median sacral vessels
    d. Right common iliac artery
    e. Right common iliac vein
A

Answer: C

There are multiple techniques for performing LIF. However, the L5/S1 level is particularly suitable for the ALIF approach due to the efficient vascular access below with bifurcation of the aorta and inferior vena cava. Hence, the L5/S1 discectomy with an anterior approach is suggested as the choice of treatment.

Supine position. For the L5/S1 exposure, a transverse incision (mini-Pfannenstiel) is performed between the umbilicus and the symphysis pubis. Dissection of skin and soft tissue is done with the diathermy with an inferior and superior flap raised to give the vertical exposure. The exposed linea alba is divided using monopolar diathermy. Tissue forceps are used to elevate and retract the left sided rectus muscles so that the retroperitoneal plane can be entered.
The retroperitoneum is approached with blunt dissection, the inferior epigastric vessels are visualized, preserved and retracted anteriorly. The psoas muscle and the genitofemoral nerve are visualized. As the vessels are identified (left common iliac artery and vein), a low profile narrow ring-based retractor blade system is positioned. The iliac arteries and veins are then exposed and retracted laterally to reveal the L5/S1 disc space, with the median sacral vessels double clipped and divided.

How well did you know this?
1
Not at all
2
3
4
5
Perfectly
9
Q
  1. A 16 year old boy presents to your clinic with a background of spastic cerebral palsy affecting all 4- limbs. He is level V on GMFCS and has a left thoracolumbar scoliosis of approximately 55degrees on XR. These also demonstrate pelvic obliquity, He has problems with sitting and recurrent lower respiratory tract infections. There is some curve flexibility on elevation.
    What is the best course of action in this scenario?

a. Anterior release and posterior correction with instrumented fusion T3-pelvis
b. Moulded brace and wheelchair modifications
c. Observation and further review in 6 months
d. Posterior correction with instrumented fusion T3-pelvis
e. Selective dorsal rhizotomy

A

Answer: D

Cerebral palsy is a common congenital condition that occurs due to insult to the immature brain presenting with cognitive and musculoskeletal abnormalities of varying severity. Non-progressive UMN disease (static encephalopathy) due to insult to immature brain.
Diagnosis is made clinically with evaluation of developmental milestones, cognitive function, and musculoskeletal abnormalities including spasticity, loss of motor control, and impaired balance.
Treatment involves a multidisciplinary approach to address spasticity, orthopedic manifestations, and cognitive function.

Orthopaedic manifestations:
- Contractures, fractures, upper extremity deformities, hip subluxation and dislocation, spinal deformity, foot deformity and gait disorders.

Pathoanatomy: mix of weakness and spasticity. Encephalopathy is static while the affected portion of the MSK changes with growth.

Primary: Abnormal tone, loss of motor control, impaired balance, spasticity.

Secondary: Contractures - start as dynamic, and become static with time (continuous muscle contraction results in shortening) and growth (growth of bones occurs at a faster longitudinal rate than muscles in spastic cerebral palsy).
Hip subluxations and dislocations, spinal deformity, foot deformity, gait deformities, fractures (low BMD)

Physiologic Classification:
- Spastic: commonest - velocity dependent increased muscle tone and hyperreflexia with slow, restricted movement due to simultaneous contraction of agonist and antagonist muscles - most amenable to operative treatments.

Athetoid: characterised by a constant succession of slow, writhing, involuntary movements.

Ataxic: characterised by inability to coordinate muscle movements -> unbalanced, wide based gait.

Mixed: Usually mixed spastic and athetoid features, involves the entire body.

Hypotonic: usually precedes spastic or ataxic for 2-3 years.

Anatomic Classification:
Quadriplegic: total body involvement and non-ambulatory
Diplegic: legs»arms but still ambulatory. IQ may be normal (injury in brain is midline)
Hemiplegic: arms and legs on one side of the body, usually with spasticity. Will be able to walk, regardless of treatment.

GMFCS:
I: Near normal gross motor function, independent ambulator
II: Walks independently, but difficulty with uneven surfaces, minimal ability to jump
III: Walks with assistive device
IV: Severely limited walking ability, primary mobility is wheelchair
V: Non-ambulator with global involvement, dependent in all aspects of care.

Spinal Disorders in Cerebral Palsy are thought to be caused by muscle weakness and truncal imbalance and most commonly present with progressive scoliosis - 20% incidence in CP. Most severe CP, higher the likelihood of scoliosis - spastic quadriplegia at highest risk, approaches 100% for bedridden children, rare in those that ambulate.

Pelvic obliquity causes deforming forces on spine and scoliosis.

CP curves are more likely to progress, begins at an earlier age, tends to be a long, stiff C-shaped curve (left sided curves are not uncommon), curve tends to have a greater sagittal plane deformity (kyphotic or lordotic), associated with pelvic obliquity, skeletal maturity is delayed in CP, bracing is less effective, longer fusions to the pelvis are often necessary, patients are more medically fragile and MDT approach is often necessary.

Goals of Surgery:
- obtain painless solid fusion with well corrected, well balanced spine with level pelvis
- decision to proceed with surgery must include careful assessment of family’s goals and careful risk-benefit analysis

Much higher complication rate in anterior surgery in CP than idiopathic scoliosis.

How well did you know this?
1
Not at all
2
3
4
5
Perfectly
10
Q
  1. A 33 year old woman is injured in an RTA and brought to ED resus. ATLS protocol is followed. No active haemorrhage is identified, but she is bradycardic and hypotensive, therefore an IV fluid challenge is administered and she is catheterised. She is unable to move either lower limb and has loss of torso sensation at and below the level of the 5th intercostal space. Sensory and motor function are intact in the upper limbs. There is no anal tone when tugging on the catheter.
    What ASIA classification does this scenario describe?

a. A
b. B
c. Can’t determine at present
d. D
e. E

A

Answer: C

How well did you know this?
1
Not at all
2
3
4
5
Perfectly
11
Q
  1. A 54 year old woman with a history of early menopause presents with kyphosis resulting from a combination of osteoporosis and degenerative disc disease. Conservative management of her back pain has failed. She is therefore offered a sagittal correction spinal procedure. Which of the following is true of subtraction osteotomy?
    a. Anterior column is opened, middle column is somewhat closed and posterior column is closed.
    b. Correction occurs at the level of the vertebral body, not the disc
    c. Classically done at T11/12 vertebrae
    d. Allows a greater degree of correction than vertebral column resection or Smith Peterson osteotomy
    e. Associated with fewer complications than vertebral column resection or Smith-Peterson osteotomy
A

Answer: B

Sagittal plane imbalance is defined as radiographic sagittal imbalance of >5cm.

Degenerative scoliosis results from the asymmetric degeneration of disc space and/or facet joints in the spine - may occur in the coronal plane (scoliosis) or sagittal plane (kyphosis/lordosis)
Factors contributing to sagittal plane imbalance:
- Osteoporosis
- Pre-existing scoliosis
- Iatrogenic instability
- Degenerative disc disease

Goals of surgery:
- Restore balance: sagittal plane balance is most reliable predictor of clinical symptoms post-operatively - can be measured by C7 plumb line. Correction of sagittal plane deformity requires intense preoperative planning. Correct lumbar lordosis to normal anatomic range:
PI = LL+/- 9°
LL ≤ 45° - TK - PI
- Most predictive of sagittal plane correction maintenance.
- Relieve pain
- Obtain solid fusion

Worse outcomes associated with baseline depression and obesity.

Osteotomies - used to regain sagittal balance in severe angulation deformities.
30 deg or more correction can be obtained through Smith-Peterson or pedicle subtraction osteotomies.

Smith-Peterson Osteotomy
- Indicated in mild-moderate sagittal imbalance requiring correction of up to 10 deg per level of osteotomy.
Pre-requisites:
- No anterior fusion at the level of the osteotomy
- Adequate correction requires adequate disc height and mobility - CORRECTION IS AT THE LEVEL OF THE DISC, therefore more correction can be obtained in the lumbar spine, where discs have more height and mobility, than the thoracic spine (less disc height and mobility).

Pedicle Subtraction Osteotomy:
Indicated in severe sagittal imbalance >12cm, requiring correction of 30-35 deg in the lumbar spine, and 25 deg in the thoracic spine.
Where anterior fusion is present - AS CORRECTION IS AT THE LEVEL OF THE VERTEBRAL BODY, NOT THE DISC.

Complication rate is significantly higher when osteotomies, revision procedures and combined anterior/posterior approaches are used.
Pseudoarthrosis, dural tear, infection, implant complication, neurologic deficits, epidural haematoma, PE, DVT, Death.

How well did you know this?
1
Not at all
2
3
4
5
Perfectly
12
Q
  1. A 42 year old male develops pain and a popping sensation in his upper right arm when catching a heavy box. Other than the likely structure injured, which of the following resides in the interval shown in the image?

a. Axillary nerve
b. Coracohumeral ligament
c. Supraspinatus tendon
d. Middle glenohumeral ligament
e. Short head of biceps

A

Answer: B

How well did you know this?
1
Not at all
2
3
4
5
Perfectly
13
Q
  1. A 56 year old lady has presented to your clinic with planovalgus deformity of her foot. What would you do to test the function of her tibialis anterior?

a. Place the patient’s ankle in plantar flexion and ask her to invert her foot against resistance
b. Place the patient’s ankle in maximal dorsiflexion and ask her to invert her foot against resistance
c. Place the patient’s ankle in maximum dorsiflexion and ask her to resist your attempt to plantarflex
d. Place the patient’s ankle in a neutral position and resisted eversion performed
e. Place the patient’s ankle in plantar flexion and eversion to maintain this position

A

Answer: A

How well did you know this?
1
Not at all
2
3
4
5
Perfectly
14
Q
  1. Which of the following is true regarding ankle arthroscopy?
    a. The anteromedial port is located lateral to the tibialis anterior tendon
    b. The anterolateral port is located just lateral to peroneus tertius tendon
    c. The superficial peroneal is at risk on placing the anteromedial portal
    d. The saphenous nerve and vein are at risk on placing the anterolateral portal
    e. An external traction device is rarely used to distract the tibiotalar joint
A

Answer: B

Portals:
- Anteromedial: primary viewing portal (typically established first) and gives access to anteromedial joint.
 Medial to tibialis anterior
 Lateral to medial malleolus
 Make portal between tibialis anterior and saphenous vein
• Anterolateral: gives access to anterolateral joint
 Just lateral to peroneus tertius and superficial peroneal nerve
 Medial to lateral malleolus
 can trace out superficial peroneal nerve prior to incision

Answer: The anterolateral port is located just lateral to peroneus tertius tendon

How well did you know this?
1
Not at all
2
3
4
5
Perfectly
15
Q
  1. With regards to the diagnostic performance of a clinical investigation

Which of the following definitions is correct?

A. Accuracy is calculated by dividing all the positive test results by all the negative test results
B. Negative predictive value is determined by dividing the true negatives by all the negative test results
C. Positive predictive value is the ability of a test to detect all cases of a disease
D. Sensitivity is the ability to identify true negatives
E. Specificity is the calculated by dividing all the true positives by all the true positives plus the false negatives.

A

Answer: B

How well did you know this?
1
Not at all
2
3
4
5
Perfectly
16
Q
  1. Calcium hydroxyapatite is the most abundant inorganic component of bone.
    What is the chemical composition of calcium hydroxyapatite?

a. Ca2(PO4)6(OH)2
b. Ca10(PO4)6(OH)5
c. Ca10(PO4)6(OH)2
d. CaPO4OH2
e. Ca2(PO4)10(OH

A

Answer: C
Ca10(PO4)6(OH)2

How well did you know this?
1
Not at all
2
3
4
5
Perfectly
17
Q
  1. During the posterior approach to the shoulder, excessive retraction of medial muscles results in injury to an artery. There is concern that an associated nerve injury has been sustained.
    What is the defect following injury to this nerve?

a. Abduction of the shoulder
b. Abduction and external rotation of the shoulder
c. Adduction and internal rotation of the shoulder
d. Extension of the shoulder
e. Internal rotation of the shoulder

A

Answer: B
Dangers:
Suprascapular nerve
passes around the base of the scapular spine (do not retract infraspinatus too vigorously)

Axillary nerve
runs through the quadrangular space beneath the teres minor (stay superior to the teres minor)
this is accompanied by the posterior circumflex humeral artery

This approach is infrequently used
this approach offers access to the posterior and inferior aspects of the shoulder
Indications
proximal humerus fracture-dislocations
glenoid fractures/osteotomy
removal loose bodies
irrigation and debridement of septic joint
scapular neck fractures

Internervous plane
teres minor (axillary n.)
infraspinatus (suprascapular n.)

How well did you know this?
1
Not at all
2
3
4
5
Perfectly
18
Q
  1. Which of the following changes occurs within articular cartilage during ageing?

a. Decreased permeability
b. Decrease in chondrocyte size
c. Increased keratan sulfate concentration
d. Increased proteoglycan synthesis
e. Increased water content

A

Answer: C

How well did you know this?
1
Not at all
2
3
4
5
Perfectly
19
Q
  1. In patients with high risk of failure of ACL reconstruction, which of the following can reduce the graft rupture of hamstring tendon autograft reconstruction?
    a. Using screws for tibial and femoral fixation
    b. Using double bundle ACL reconstruction technique
    c. Using a knee brace for 6 weeks post-op
    d. Doing a lateral extra-articular tenodesis in combination with ACL reconstruction
    e. Using all inside ACL reconstruction technique
A

Answer: D

ACL: femoral attachment – lateral intercondylar ridge demarcates the anterior edge of the ACL, with the bifurcate ridge separating the anteromedial and posterolateral bundle attachment. Tibial attachment – anterior tibia, between intercondylar eminences.
90% type I collagen, 10% type III collagen.
Blood supply: middle geniculate artery, Innervation: posterior articular nerve (branch of tibial nerve). 2200N strength (anterior).
Provides 85% of stability to prevent anterior translation of the tibia relative to the femur, and a secondary restraint to tibial rotation and varus/valgus rotation.
Lachman’s test = most sensitive examination test for ACL – predominantly assesses the anteromedial bundle of ACL fibres.
- Grading: A= firm endpoint, grade B= no endpoint
o Grade 1: <5mm translation
o Grade 2 A/B: 5-10mm translation
o Grade 3 A/B: >10mm translation
PCL tear can give false Lachman due to posterior subluxation.
Pivot Shift: extension to flexion – reduces at 20-30 degrees of flexion (patient must be completely relaxed). Predominantly assesses the posterolateral bundle of ACL fibres.
Quadriceps autograft shown to have equivalent outcomes to hamstring and patella tendons.
Allograft lacks donor site morbidity but have increased cost and increased failure rates in the young population.

Single vs Double Bundle:
- Single is most common, double may restore native knee kinematics with less laxity – no PROMs between the two.

Revision ACL reconstruction:
• approach considerations
o cause for prior ACL failure – graft failure for any cause is approximately 5%, tunnel malposition is the commonest cause (70% failures)
o concomitant pathology
o prior graft selection
o careful assessment of the underlying cause of re-rupture
• technique
o high strength grafts (quad tendon, hamstring, allograft)
o dual or back-up fixation (suspension + interference screws)
o bone grafting and reconstruction in cases of previous tunnel dilation (15mm) or if interfering with anatomic tunnel creation
o addition of anterolateral ligament/ALL reconstruction (lateral extra-articular tenodesis) controversial
o re-harvesting BPTB (bone-patella-tendon-bone) is contraindicated

Lateral extra-articular tenodesis (LET) to augment ACL reconstruction: The anterolateral structures (anterolateral ligament and iliotibial band) of the knee joint are important for providing rotational stability to the knee. In patients who have ACL tears with disruption to the anterolateral corner of the knee, ACLR alone may leave residual rotational instability and increase the risk of ACL re-rupture.
To decrease failure rates following ACLR, patients with ligamentous laxity, hyperextension, strongly positive pivot shift test, and those in elite competitive pivoting sports, particularly at a younger age, may undergo ACLR with extra-articular tenodesis. The most commonly performed extra-articular augmentation procedures with ACLR include lateral extra-articular tenodesis (LET) and anterolateral ligament reconstruction (ALLR).
LET is reported to offload ACL graft by an average of 43%, reducing the risk of graft stretching or re-rupture during the rehabilitation phase.
In young patients engaging in pivoting sports, ACL reconstruction with ALLR is reported to have a 2.5 times reduction in graft failure rate than BPTB autograft alone and 3.1 times reduction in graft failure rate compared to HT autograft alone, at a mean of 38 months follow-up. Patients with ligamentous hyperlaxity undergoing ACL reconstruction with HT autograft and ALLR were found to have a lower graft failure rate and improved knee stability, compared to isolated HT autograft ACLR, 3.3% vs. 21.7%, at mean follow-up of 28.1 vs. 29.6 months respectively.

Therefore answer:
Doing a lateral extra-articular tenodesis in combination with ACL reconstruction

How well did you know this?
1
Not at all
2
3
4
5
Perfectly
20
Q
  1. Which of these structures does not cause compression of the ulnar nerve around the elbow?

a. Medial epicondyle
b. Osbourne’s ligament
c. Arcade of Struthers
d. Arcade of Frohse
e. FCU aponeurosis

A

Answer: D

How well did you know this?
1
Not at all
2
3
4
5
Perfectly
21
Q
  1. A 60 year old man presents with symptoms of worsening lower back, buttock and right lower limb pain. This has persisted several months despite physiotherapy and activity modifications. On examination she has normal sensation in the right lower limb, but 4/5 power in the right EHL and tibialis anterior. MRI shown. Which treatment option is most likely to give the best long-term results?

a. CT guided cyst aspiration
b. L4/5 laminectomy, cyst excision and complete facetectomy
c. L4/5 laminectomy and cyst excision
d. L4/5 posterior instrumented fusion, laminectomy and cyst excision
e. L4/5 posterior un-instrumented fusion, laminectomy and cyst excision

A

Answer: D

How well did you know this?
1
Not at all
2
3
4
5
Perfectly
22
Q
  1. An isthmic spondylolisthesis is the result of a defect in which part of the vertebra?

a. Lamina
b. Facet
c. Pars interarticularis
d. Transverse process
e. Mammillary process

A

Answer: C

How well did you know this?
1
Not at all
2
3
4
5
Perfectly
23
Q
  1. In congenital scoliosis, which anatomic pattern carries the worst prognosis?

A. Block vertebra
B. Unilateral unsegmented bar with contralateral hemivertebra
C. Segmented hemivertebra
D. Unsegmented hemivertebra
E. Unilateral unsegmented bar

A

Answer: B

How well did you know this?
1
Not at all
2
3
4
5
Perfectly
24
Q
  1. In spina bifida, a protruding sac without neural elements is classified as:

a. A gibbus
b. Spina bifida occulta
c. Rachischisis
d. Meningiocele
e. Myelomeningocele

A

Answer: D

How well did you know this?
1
Not at all
2
3
4
5
Perfectly
25
Q
  1. A 54 year old man with rheumatoid arthritis undergoes flexion-extension cervical radiographs that show evidence of atlanto-axial subluxation. This is usually related to destruction of which spinal ligament?
    a. Alar ligament
    b. Anterior longitudinal ligament
    c. Apical ligament
    d. Posterior longitudinal ligament
    e. Transverse ligament
A

Answer: E
Transverse ligament – due to pannus formation

How well did you know this?
1
Not at all
2
3
4
5
Perfectly
26
Q
  1. All the following are true regarding Coleman block test during foot and ankle examination, except:
    a. It is used to differentiate a flexible from rigid pes cavovalgus deformity
    b. It is performed by placing the heel and lateral border of the foot on a block, with the 1st ray unsupported.
    c. In a flexible deformity, the heel will correct to a neutral or valgus position.
    d. In a rigid deformity, the heel will remain in varus position
    e. A flexible deformity indicates that the deformity is forefoot driven and the subtalar joint is not stiff
A

Answer: A

Cavovarus foot is a common condition, seen in both paediatric and adult population, caused by either neurologic (67%( or traumatic disorder, and presents with a cavus arch and hindfoot varus.
Deformity is characterised by:
- Cavus (elevated longitudinal arch)
- Plantarflexion of 1st ray and pronated forefoot
- Hindfoot varus
- Forefoot adduction

Bilateral is most commonly due to Charcot Marie Tooth Disease.
Muscle imbalances generate the deformity:
- Weak tibialis anterior and peroneus brevis overpowered by strong peroneus longus and posterior tibialis. This results in a plantarflexed 1st ray and forefoot pronation with compensatory hindfoot varus.
- With the 1st metatarsal plantarflexed and forefoot pronated, the medial forefoot strikes the ground first.
- The subtalar joint supinates to bring the lateral forefoot to the ground and maintain 3-point contact, resulting in hindfoot varus.
- While initially flexible, hindfoot varus can become rigid with time.

Other conditions that present with a cavovarus foot: Charcot marie Tooth disease, Cerebral Palsy, Friedrich’s ataxia, spinal cord lesions, polio, amniotic band syndrome.

History:
- Recurrent ankle sprains and lateral ankle pain (peroneal tendon pathology)
- Lateral foot pain: excessive weight bearing by the lateral foot due to deformity (can result in 5th MT stress fractures.
- Painful plantar calluses under 1st MT head and 5th MT head or base.
- Plantar fasciitis (elevated medial arch, forefoot pronation and tight gastrocnemius lead to contracture of plantar fascia

Coleman Block Test: Evaluates flexibility of hindfoot deformity.
- Place 1inch block under the lateral foot (eliminates contribution of the plantarflexed 1st ray and forefoot pronation to the hindfoot deformity)
- A flexible hindfoot will correct to neutral or valgus when block placed under lateral aspect of foot. Rigid hindfoot will not correct to normal.

Answer: used to differentiate a flexible from rigid pes cavovalgus deformity – VARUS!!!

How well did you know this?
1
Not at all
2
3
4
5
Perfectly
27
Q
  1. A patient has femoracetabular impingement with a labral tear, in order to perform a minimally invasive CAM resection, the surgeon approached the hip through an anterior and anterolateral portal. Which of the following structures is compromised following an extensive inter-portal capsulotomy during hip arthroscopy?

a. Ischiofemoral ligament
b. Pubofemoral ligament
c. Iliofemoral ligament
d. Ligamentum teres
e. Zona orbicularis

A

Answer: C

How well did you know this?
1
Not at all
2
3
4
5
Perfectly
28
Q
  1. In 2017 the Medicines and Healthcare Products Regulatory Agency published a Medical Device Alert to standardise follow-up for patients who had a total hip replacement with a metal on metal bearing surface.

Which of the following patients would be suitable for follow-up at 1 year, 7 years then every three years.

A. ASR resurfacing, male patient, 50 head size, asymptomatic.
B. BHR resurfacing, female patient, 50 head size, asymptomatic.
C. THR, male patient, 32 head size, asymptomatic.
D. BHR resurfacing, male patient, 50 head size, asymptomatic.
E. BHR resurfacing, male patient, 50 head size, symptomatic.

A

Answer: D

How well did you know this?
1
Not at all
2
3
4
5
Perfectly
29
Q
  1. What is the most lateral tendon within the anterior compartment of the leg?

a. Peroneus tertius
b. Peroneus brevis
c. Tibialis anterior
d. Extensor hallucis longus
e. Extensor digitorum longus

A

Answer: A

How well did you know this?
1
Not at all
2
3
4
5
Perfectly
30
Q
  1. When operating upon a distal tibia non-union, current guidance suggests 2 specimens be sent to pathology looking for evidence of infection. How many should be sent for microbiology?

a. 3
b. 5
c. 7
d. 6
e. 4

A

Answer: B

How well did you know this?
1
Not at all
2
3
4
5
Perfectly
31
Q
  1. For which of the below indications is ankle distraction arthroplasty most appropriate?

a. Inflammatory arthritis
b. Post traumatic OA
c. AVN talus
d. Talar OCL
e. Syndesmotic degeneration

A

Answer: B

How well did you know this?
1
Not at all
2
3
4
5
Perfectly
32
Q
  1. A plantar flexion malunion through a TMT joint fusion manifests clinically with plantar forefoot pain. Which is the most appropriate initial treatment

A. Orthotic insole
B. Revision fusion to correct malunion
C. Achilles lengthening
D. Supramalleolar osteotomy
E. Excision of metatarsal heads

A

Answer: A

How well did you know this?
1
Not at all
2
3
4
5
Perfectly
33
Q
  1. Within rough endoplasmic reticulum during the early stages of collagen synthesis, post translational modification occurs, where the single alpha chain forms a triple helix (pro-collagen). One of the enzyme dependent processes requires the cofactor vitamin C.
    What is this process called?

a. Hydroxylation
b. Glycosylation
c. Exocytosis
d. Trimming terminal ends
e. Cross linking (covalent bond forming)

A

Answer: A

How well did you know this?
1
Not at all
2
3
4
5
Perfectly
34
Q
  1. All of the following are true regarding operative fixation for diabetic patients with ankle fractures, EXCEPT:

a. Is associated with higher rate of complications such as infections, wound complications and delayed union.
b. Adoption of a ‘super construct’ approach that includes extending fixation beyond the immediate zone of injury is considered beneficial
c. Patients may also require a prolonged period of immobilisation
d. Adoption of minimally invasive approaches and minimal metalwork is considered beneficial
e. Close follow up helps to early diagnosis of Charcot arthropathy development

A

Answer: D

How well did you know this?
1
Not at all
2
3
4
5
Perfectly
35
Q
  1. A six year old boy who loves to play cricket complains of elbow pain, XR shows no loose bodies. Clinical suspicion is this may be due to repetitive throwing action and requests an MRI, what is the most likely diagnosis?

a. Osteochondritis dissecans
b. Kienbock’s disease
c. Kohler’s disease
d. Severs’ disease
e. Panner Disease

A

Answer: E

How well did you know this?
1
Not at all
2
3
4
5
Perfectly
36
Q
  1. All are true regarding Hawkins sign, Except

A. presence of the sign means that the patient is unlikely to develop avascular necrosis (AVN) of the talus.
B. identified as a radiolucent line of the talar dome.
C. absence of the sign means that the patient will develop AVN.
D. is a good prognostic sign for this injury.
E. appears on the x-ray 6–8 weeks following the injury.

A

Answer: C

How well did you know this?
1
Not at all
2
3
4
5
Perfectly
37
Q
  1. A middle-aged female patient presents with medial hindfoot pain and a swollen ankle. Which of the following is true?

A. Inability to perform heel raise is confirmatory of tibialis posterior tendon dysfunction.
B. An ability to perform single heel raise confirms an intact tibialis posterior tendon.
C. While performing a single heel raise a false-positive result can be elicited by the patient leaning forward.
D. Tibialis posterior tendon strength can be assessed by asking the patient to forcefully invert the plantigrade ankle.
E. Initial management may require a short period of cast immobilisation.

A

Answer: E

How well did you know this?
1
Not at all
2
3
4
5
Perfectly
38
Q
  1. A young patient presents with recurrent ankle instability and non-healing 5th metatarsal fracture. Which of the following is true?

A. A positive peek-a-boo sign will help to guide further management options.
B. A CT scan should be performed to assess the state of healing of the 5th metatarsal fracture.
C. The patient should be offered surgical fixation of the non-healing fracture.
D. Surgery has no role in the management of recurrent ankle instability.
E. It is unnecessary to enquire of a positive family history.

A

Answer: A

How well did you know this?
1
Not at all
2
3
4
5
Perfectly
39
Q

A 67 year old woman reports a 3 month history of worsening back pain. Over the last 36 hours she has developed progressive weakness in both lower limbs which has increasingly impacted on her walking. She is now not able to ambulate, though prior to this she mobilised unaided. She complains of not being able to void urine. On neurological examination, she has 3/5 power throughout both lower limbs, and a sensory level corresponding to the pathology seen in the MRI image of her spine. She has a staging CT which shows likely breast cancer with metastases to the spine, left proximal femur, as well as lesions in the lungs and liver.
Which of the following is the most appropriate step in management of the patient?

a. Bone biopsy
b. Decompression and stabilisation
c. Decompressive laminectomy
d. En bloc excision and reconstruction
e. Palliative radiotherapy based on prognosis

A

Answer: B

How well did you know this?
1
Not at all
2
3
4
5
Perfectly
40
Q

. An 82 year old woman presents 10 days after a fall from standing height, due to progressively worsening neck pack. She describes no weakness or pain in her arms, nor any changes from her usual dexterity. CT image shown (Type II odontoid peg fracture) . Which of the following would you explain that she is at increased risk of with conservative as opposed to surgical management?

a. Cervical radiculopathy
b. Deterioration in dexterity
c. Dysphagia
d. Fracture non-union
e. Myelopathy

A

Answer: D

How well did you know this?
1
Not at all
2
3
4
5
Perfectly
41
Q
  1. An elderly patient presents with forefoot pain. Examination reveals asymptomatic hallux valgus deformity, plantar callosity and painful 2nd and 3rd metatarsophalangeal joints.
    Which of the following is true?

a. The hallux valgus deformity is an incidental finding
b. Plantar callosity confirms the need for surgical intervention
c. It is useful to flex the knee as part of the clinical examination
d. Improvement in ankle dorsi-flexion with passive flexion is an indication for surgery
e. Metatarsal shortening osteotomy would be essential in managing this patient

A

Answer: C

How well did you know this?
1
Not at all
2
3
4
5
Perfectly
42
Q

. Authors conducted a randomised trial to compare the effectiveness of physiotherapy vs arthroscopic decompression for rotator cuff tendinopathy. The results were analysed using intention to treat. Which of the following is true?

A. Intention to treat analysis was based on patients’ intention to be treated.
B. Patients were analysed only if they completed the allocated treatment.
C. Intention to treat would allow a pragmatic analysis of the effectiveness of the intervention.
D. Patients who crossed-over to the alternative intervention were excluded from analysis.
E. The results could be affected by confounding bias.

A

Answer: C

How well did you know this?
1
Not at all
2
3
4
5
Perfectly
43
Q
  1. A 63 year old man with single-level degenerative lumbar stenosis undergoes an ALIF (anterior lumbar interbody fusion) with a bone graft substitute to aide osteoinduction. Which rhBMP (recombinant human Bone Morphogenic Protein) is licensed for this?

a. 2
b. 3
c. 4
d. 6
e. 7

A

Answer: A

How well did you know this?
1
Not at all
2
3
4
5
Perfectly
44
Q

. Which of the following is not associated with osteonecrosis of the humeral head after a proximal humerus fracture?

a. Intact calcar length of 6mm
b. Surgical neck displacement 12mm
c. 4-part with head splitting fracture pattern
d. Medial hinge disruption
e. Greater tuberosity displacement of 6mm

A

Answer: E

How well did you know this?
1
Not at all
2
3
4
5
Perfectly
45
Q
  1. A 29 year-old-man has brachialgia with weak wrist extension. What MRI findings are consistent with this

A. C4/5 central disc
B. C5/6 foraminal disc
C. C6/7 central disc
D. C6/7 foraminal disc
E. C7/T1 central disc

A

Answer: B

Nerve root anatomy
key differences between cervical and lumbar spine are
pedicle/nerve root mismatch
cervical spine C6 nerve root travels above C6 pedicle (mismatch)
lumbar spine L5 nerve root travels under L5 pedicle (match)
extra C8 nerve root (no C8 pedicle) allows transition

horizontal (cervical) vs. vertical (lumbar) anatomy of nerve root
because of vertical anatomy of lumbar nerve root, a paracentral and foraminal disc will affect different nerve roots, whereas in the cervical spine both foraminifera and central discs will affect the same nerve root

C4 radiculopathy
scapular winging
numbness and pain at the base of the neck
C5 radiculopathy
deltoid and biceps weakness
diminished biceps reflex
pain and numbness in the superior shoulder and lateral upper arm
C6 radiculopathy
brachioradialis and wrist extension weakness
diminished brachioradialis reflex
paresthesias in the thumb and radial arm
C7 radiculopathy
triceps and wrist flexion weakness
diminished triceps reflex
paresthesia in the middle finger
most commonly affected nerve root in cervical radiculopathy in several studies
C8 radiculopathy
weakness to distal phalanx flexion of middle and index finger (difficulty with fine motor function)
paresthesias in ring and little finger
C8 radiculopathy is extremely rare and often manifests similarly as ulnar neuropathy
T1 radiculopathy
intrinsic hand muscle weakness
axillary numbness
ipsilateral Horner’s syndrome

because of horizontal anatomy of cervical nerve root, a central and foraminal disc will affect the same nerve root

How well did you know this?
1
Not at all
2
3
4
5
Perfectly
46
Q
  1. A 70 year old man with back and leg pain was weak resisted ankle dorsiflexion. What finding would be present on MRI?

a. L3/4 extraforaminal stenosis
b. L3/4 foraminal stenosis
c. L3/4 lateral recess stenosis
d. L4/5 central stenosis
e. L4/5 Lateral recess stenosis

A

Answer: C

How well did you know this?
1
Not at all
2
3
4
5
Perfectly
47
Q
  1. A 15 year old girl has a pain thoracolumbar scoliosis, worse at night and relieved by ibuprofen. What would you see on histology?
    a. Anaplastic spindle cells in storiform pattern
    b. Anastomosing trabeculae with osteoblastic rimming
    c. Round blue cells in sheets
    d. Hyperchromic epithelial cells
    e. Irregular woven bone
A

Answer: B

Pathological scoliosis is a form of scoliosis resulting from a benign bone tumour, most commonly located in the posterior elements of the spine.
Causes:
- Osteoid osteomas: occur in the apex of the concavity of the curve, curves are typically rigid, can occur in the vertebral body or posterior elements
o XR cortical thickening radiolucent nidus (osteoid osteoma <2cm in diameter, osteoblastoma > 2cm)
- Osteoblastomas: larger lesion than osteoid osteoma, pain is usually less severe than osteoid osteoma (less relief with anti-inflammatories)
Scoliosis is thought to develop in response to painful paraspinal muscle spasms.
Surgical management only indicated in painful, progressive scoliosis.

Answer: Anastomosing trabeculae with osteoblastic rimming (osteoid osteoma)

How well did you know this?
1
Not at all
2
3
4
5
Perfectly
48
Q
  1. Anterior cervical approach is performed in a myelopathic patient. In a lower cervical approach, a muscle crosses the operative field and can be divided.
    Which nerve supplies this muscle?

a. Ansa cervicalis
b. Cranial nerve VII
c. Cranial nerve X
d. Cranial nerve XI
e. Recurrent laryngeal nerve

A

Answer: A

How well did you know this?
1
Not at all
2
3
4
5
Perfectly
49
Q
  1. During the anterior approach to the cervical spine, a deep structure is reflected sub-periosteally and laterally each side of the midline to immediately expose the anterior vertebral body. What is this structure?
    a. Longus capitis
    b. Longus coli
    c. Multifidus
    d. Prevertebral fascia
    e. Sternocleidomastoid
A

Answer: B

How well did you know this?
1
Not at all
2
3
4
5
Perfectly
50
Q
  1. A 16 year old boy presents with a painful heel of one month duration. Examination reveals soft tissue swelling and warmth to the plantar aspect of his heel. Radiographs show a sclerotic mass with periosteal reaction. MRI T1 imagin shows a heterogenous low signal mass which enhanced during contrast administration. What is the most likely diagnosis?

A. Chondrosarcoma
B. Ewings Sarcoma
C. Fibrosarcoma
D. Metastatic Lesion
E. Osteosarcoma

A

Answer: E

How well did you know this?
1
Not at all
2
3
4
5
Perfectly
51
Q
  1. Ultra-high-molecular weight polyethylene is manufactured to provide a low friction bearing surface that is resistant to wear. One of the evolutions in manufacturing is adding Vitamin E to soak up free radicals. Select the process that leads to polyethylene degradation:

a. Condensation polymerisation
b. Annealing
c. Oxidation
d. Gamma irradiation
e. Vacuum packing

A

Answer: C

52
Q
  1. A prospective randomised trial is conducted to test the efficacy of conservative treatment vs arthroplasty for 3 and 4 part proximal humerus fractures. To determine statistical significance when comparing height between these two treatment groups, which statistical test would you use?

a. One-way ANOVA
b. Student T-test
c. Spearman rank correlation
d. Chi-squared
e. Paired T-test

A

Answer: B

A t-test is any statistical hypothesis test in which the test statistic follows a Student’s t-distribution under the null hypothesis. It is most commonly applied when the test statistic would follow a normal distribution if the value of a scaling term in the test statistic were known.

Two-sample t-test is used when the data of two samples are statistically independent, while the paired t-test is used when data is in the form of matched pairs.

53
Q
  1. Which protein- coding gene is the master regulator in the control of proliferating chondrocytes in the developing embryo?

a. SRY-Box Transcription Factor 8 (SOX-8)
b. Collagen 2 (COL2A1)
c. SRY-Box Transcription Factor 9 (SOX-9)
d. Collagen 10 (COL10A1)
e. Aggrecan (ACAN)

A

Answer: C

Cell differentiation
cartilage is formed from mesenchymal stem cells designated towards the cartilagenous lineage
multi-step process involving activation and migration of cells to necessary sites
SOX-9 is a key transcription factor involved in the differentiation of cells towards the cartilage lineage

Cell biology
endochondral bone formation occurs with a cartilage model
chondrocytes produce cartilage which is absorbed by osteoclasts
osteoblasts lay down bone on cartilaginous framework (bone replaces cartilage, cartilage is not converted to bone)
forms primary trabecular bone
bone deposition occurs on metaphyseal side
type X collagen associated with endochondral ossification
Molecular biology
chondrocytes play a critical role in endochondral bone formation throughout the formation of the cartilage intermediate
transcription factors involved in regulation of chondrocytes include
Sox-9
considered a major regulator of chondrogenesis, regulates several cartilage-specific genes during endochondral ossification, including collagen types II, IV, and XI and aggrecan
PTHrP
delays differentiation of chondrocytes in the zone of hypertrophy

54
Q
  1. A 45 year old patient attends ED with medial ankle pain. She is found to have a valgus hind foot deformity of 15 degrees and a flexible flat foot deformity. She has failed conservative management. What is the surgical option for this patient?
    a. Flexor digitorum longus transfer to tibialis posterior tendon
    b. Forefoot correction osteotomy
    c. Lateral column lengthening osteotomy
    d. Medial displacement calcaneal osteotomy
    e. Triple arthrodesis
A

Answer: D

medial displacement calcaneal osteotomy (MDCO)
used in stage IIA (insignificant forefoot abduction)

Flexible Pes Planovalgus, also known as Flexible Flatfoot, is a common idiopathic condition, caused by ligamentous laxity that presents with a decrease in the medial longitudinal arch, a valgus hindfoot and forefoot abduction with weight-bearing.
Diagnosis can be made clinically with a foot that is flat with standing and reconstitutes with toe walking, hallux dorsiflexion, or foot hanging.
Treatment is usually observation, and stretching with majority of cases resolving over time. Rarely, surgical management is indicated for patients with progressive deformities that do not resolve with nonoperative management.

Symptoms
usually asymptomatic in children
may have arch pain or pretibial pain

Physical exam
inspection: foot is only flat with standing and reconstitutes with toe walking, hallux dorsiflexion, or foot hanging
valgus hindfoot deformity
forefoot abduction

Differential
Tarsal coalition
Congenital vertical talus
Accessory navicular

Posterior Tibial Tendon Insufficiency is the most common cause of adult-acquired flatfoot deformity, caused by attenuation and tenosynovitis of the posterior tibial tendon leading to medial arch collapse.
Diagnosis can be made clinically with loss of medial arch of the foot which may progress to hindfoot valgus, forefoot abduction and subsequent development of midfoot osteoarthritis.
Treatment is nonoperative with orthotics and ankle braces in early stages. A variety of surgical options are available and indicated for progressive and rigid deformities, subtalar or midfoot arthritis, and failure of nonoperative management.

Pathoanatomy
early disease
early tenosynovitis progresses to PTTI
leads to loss of medial longitudinal arch dynamic stabilization
late disease
PTTI contributes to attritional failure of static hindfoot stabilizers and collapse of the medial longitudinal arch
spring ligament complex (e.g., superomedial calcaneonavicular ligament)
plantar fascia
plantar ligaments
fixed degenerative joint changes occur at late stages
foot deformity
pes planus
hindfoot valgus
forefoot varus
forefoot abduction
Associated conditions
inflammatory arthropathy
tarsal coalition
young person with rigid pes planus and/or recurrent ankle sprains

tibialis posterior
originates from posterior fibula, tibia, and interosseous membrane
innervated by tibial nerve (L4-5)
Tendon
posterior tibial tendon (PTT) lies posterior to the medial malleolus before dividing into 3 limbs
anterior limb
inserts onto navicular tuberosity and first cuneiform
middle limb
inserts onto second and third cuneiforms, cuboid, and metatarsals 2-4
posterior limb
inserts on sustentaculum tali anteriorly

Biomechanics
PTT lies in an axis posterior to the tibiotalar joint and medial to the axis of the subtalar joint
functions as a primary dynamic support for the arch
acts as a hindfoot invertor
adducts and supinates the forefoot during stance phase of gait
acts as secondary plantar flexor of the ankle
major antagonist to PTT is peroneus brevis
activation of PTT allows locking of the transverse tarsal joints creating a rigid lever arm for the toe-off phase of gait

55
Q
  1. A 76 year old post-menopausal lady with previous fragility fracture attends fracture clinic for follow up. She has previously discontinued bisphosphonate treatment due to an intolerance but since her recent injury the osteoporosis nurse has recommended she starts on denosumab. The osteoprotective effects of denosumab are due to its analogous function to:

a. Receptor activator of nuclear factor-kappa B Ligand (RANK-L)
b. Tumour necrosis factor
c. Parathyroid Hormone
d. Nuclear Factor kappa- light-chain-enhancer of activated B cells (NF-KB)
e. Osteoprotegerin

A

Answer: E

Bone metabolism is a dynamic process that balances bone formation and bone resorption
central to this process is the RANK/RANKL/OPG pathway
bone formation
performed by stimulating osteoblasts and inhibiting osteoclasts
bone resorption
performed by active osteoclast
stimulated by RANKL in normal process
stimulated by PTHrP in pathologic process (metastatic disease)

Osteoblasts produce
RANKL
binds RANK and stimulates osteoclastic bone resorption
osteoprotegerin (OPG)
inhibits osteoclast differentiation, fusion, and activation
decoy receptor produced by osteoblasts and stromal cells that binds to and sequesters RANKL
alkaline phosphatase

Osteoclast Inhibition decreases bone resorption
Molecules that inhibit bone resorption
osteoprotegerin (OPG)
calcitonin
interacts directly with the osteoclast via cell-surface receptors
estrogen (via decrease in RANKL)
stimulates bone production (anabolic) and prevents resorption
inhibits activation of adenylyl cyclase
transforming growth factor beta (TGF beta) (via increase in OPG)
interleukin 10 (IL-10)
suppresses osteoclasts

Osteoclast activation stimulates bone resorption
Molecules that stimulate bone resorption
RANKL
RANKL (ligand) is secreted by osteoblasts and binds to the RANK receptor on osteoclast precursor and mature osteoclast cells
PTH
activation of its receptor stimulates adenylyl cyclase
binds to cell-surface receptors on osteoblasts to stimulate production of RANKL and M-CSF
interleukin 1 (IL-1)
stimulates osteoclast differentiation and thus bone resorption
1,25 dihydroxy vitamin D
stimulates RANKL expression
prostaglandin E2
activates adenylyl cyclase and stimulates resorption
IL-6 (myeloma)
MIP-1A (myeloma)

Osteoporosis
can result from loss of function of the OPG gene, leading to constitutive activation of osteoclasts which results in uncontrolled bone resorption and ultimately leads to osteoporosis
Osteopetrosis
condition caused by a genetic defect resulting in absence of osteoclastic bone resorption
a mouse RANKL knockout model creates a osteopetrosis-like condition
Paget disease
felt to be caused by alterations in cytoplastmic binding to RANK or mutations in the OPG gene
Osteolytic bone metastasis
found to be mediated by the RANK and RANKL pathway
RANKL is produced directly by the cancer cells
blocking of RANKL by OPG results in decreased skeletal metastasis in animal models
bisphosphonates decrease skeletal events in cancer metastasis
Osteolysis following joint arthroplasty
polyethylene wear debris is phagocytized by macrophage leads to activation of the macrophage
additional macrophages are recruited with release of additional cytokines including RANKL
RANKL activates osteoclasts which leads to bone resorption around implants

56
Q

. Which these muscle group and muscle contraction combination is correct for the initial contact phase of gait?

a. Calf muscles / Eccentric
b. Gluteus maximus / Concentric
c. Hamstrings / isometric
d. Pre-tibial muscles / Eccentric
e. Quadriceps / Concentric

A

Answer: D

One gait cycle is measured from heel-strike to heel-strike
consists of
stance phase
period of time that the foot is on the ground
~60% of one gait cycle is spent in stance
during stance, the leg accepts body weight and provides single limb support
swing phase
period of time that the foot is off the ground moving forward
~40% of one gait cycle is spent in swing
the limb advances
Stride is the distance between consecutive inital contacts of the same foot with the ground
Step is the distance between initial contacts of the alternating feet

Stance phase
Initial contact (heel strike)
definition
occurs when foot contacts the ground
muscular contractions
hip extensors contract to stabilize the hip
quadriceps contract eccentrically
tibialis anterior contracts eccentrically
Loading response (initial double limb support)
marks the beginning of the initial double limb stance
definition
occurs after initial contact until elevation of opposite limb
bodyweight is transferred on to the supporting limb
muscular contractions
ankle dorsiflexors (tibialis anterior) contract eccentrically to control plantar flexion moment
quads contract to stabilize knee and counteract the flexion moment (about the knee)
Mid-stance (single limb support)
initial period of single leg support
definition
from elevation of opposite limb until both ankles are aligned in coronal plane
muscular contractions
gluteus medius and calf muscles undergo eccentric contraction

Terminal stance (single limb support)
definition
begins when the supporting heel rises from the ground and continues until the opposite heel touches the ground
muscular contractions
toe flexors and tibialis posterior contract and are the most active during this phase
Pre-swing (second double limb support)
is the start of the second double limb stance in the gait cycle
definition
from initial contact of opposite limb to just prior to elevation of ipsilateral limb
muscular contractions
hip flexors contract to propel advancing limb

Swing phase
Initial swing (toe off)
start of single limb support for opposite limb
definition
from elevation of limb to point of maximal knee flexion
muscular contractions
hip flexors concentrically contract to advance the swinging leg
Mid-swing (foot clearance)
definition
following knee flexion to point where tibia is vertical
muscular contractions
ankle dorsiflexors contract to ensure foot clearance
Terminal swing (tibia vertical)
definition
from point where tibia is vertical to just prior to initial contact
muscular contractions
hamstring muscles decelerate forward motion of thigh

Center of gravity (COG)
in standing position is 5cm anterior to S2 vertebral body

57
Q

During the design process for an osteosynthesis device, a novel material design is considered. A material that undergoes elastic deformation, but no plastic deformation is termed:

a. Stiff
b. Viscoelastic
c. Ductile
d. Anisotropic
e. Brittle

A

Answer: E

Basic definitions
load = a force that acts on a body

stress
intensity of an internal force
calculation
force / area
units Pascal’s (Pa) or N/m2

strain
relative measure of the deformation of an object
calculation : change in length / original length
units none

Mechanical property definitions
elastic deformation
reversible changes in shape to a material due to a load
material returns to original shape when load is removed
plastic deformation
irreversible changes in shape to a material due to a load
material DOES NOT return to original shape when load is removed
toughness
definition
amount of energy per volume a material can absorb before failure (fracture)
calculation
area under the stress/strain curve
units
joules per meter cubed, J/m3
creep
increased load deformation with time under constant load
load relaxation
decrease in applied stress under conditions of constant strain
hysteresis (energy dissipation)
characteristic of viseoelastic materials where the loading curve does not follow the unloading curve
the difference between the two curves is the energy that is dissipated
finite element analysis
breaking up a complex shape into triangular or quadrilateral forms and balancing the forces and moments of each form to match it with its neighbor

Elastic zone
the zone where a material will return to its original shape for a given amount of stress
“toe region”
applies to a ligaments stress/strain curve
represents straightening of the crimped ligament fibrils
Yield point
the transition point between elastic and plastic deformation
Yield strength
the amount of stress necessary to produce a specific amount of permanent deformation
Plastic zone
the zone where a material will not return to its orginal shape for a given amount of stress
Breaking point
the object fails and breaks
Ultimate (Tensile) strength
defined as the load to failure
Hooke’s law
when a material is loaded in the elastic zone, the stress is proportional to the strain
Young’s modulus of elasticity
measure of the stiffness (ability to resist deformation) of a material in the elastic zone
calculated by measuring the slope of the stress/strain curve in the elastic zone
a higher modulus of elasticity indicates a stiffer material

Brittle material
a material that exhibits linear stress stain relationship up until the point of failure
undergoes elastic deformation only, and little to no plastic deformation
examples
PMMA
ceramics
Ductile Material
undergoes large amount of plastic deformation before failure
example
metal
Viscoelastic material
a material that exhibits a stress-strain relationship that is dependent on duration of applied load and the rate by which the load is applied (strain rate)
a function of the internal friction of a material
examples
ligaments
bone
Isotropic materials
possess the same mechanical properties in all directions
example
golf ball
Anisotropic materials
possess different mechanical properties depending on the direction of the applied load
examples
ligaments
bone

58
Q
  1. A 36 year old female sustains an iatrogenic injury to her median nerve during routine carpal tunnel decompression. Following nerve transection the cellular event signalling the start of regeneration is the:

A. Degradation of the myelin sheath
B. Schwann cell dedifferentiation
C. Formation of Büngner bands
D. Chromatolysis of the nucleus
E. Schwann cell proliferation

A

Answer: D

Axonal reaction / central chromatolysis / Wallerian degeneration: When the axon of a neuron is cut or damaged, the axon and its myelin sheath undergo degeneration distal to the lesion (Wallerian degeneration). The sequence of events that takes place in the cell body is known as central chromatolysis or axonal reaction.

59
Q
  1. You review a 19 year old male patient in outpatients who has suffered a progressive muscle weakness and has been declining recently. He has a waddling gait, positive Gower’s sign. He walks on his tip toes and you note calf enlargement. His condition does not affect anyone in his immediate family. What is his underlying diagnosis?

a. Friedrich’s ataxia
b. Duchenne’s muscular dystrophy
c. Becker muscular dystrophy
d. Charcot Marie Tooth
e. Spinal muscular atrophy

A

Answer: C

Becker Muscular Dystrophy - similar to DMD in that sex linked recessive, calf pseudo hypertrophy present, elevated CK, but differs in that dystrophin is decreased rather than absent, later onset in life with a longer life expectancy, more prone to cardiomyopathy.

DMD = dystrophin absence, young males only, dystrophin absence leads to poor muscle fibre regeneration and progressive replacement of muscle tissue with fibrous and fatty tissue, therefore skeletal and cardiac muscle lose elasticity and strength.
X-linked recessive
Xp21.2 dystrophin gene defect (point deletion, third spontaneous mutations)
Ortho manifestations: calf pseudo hypertrophy, scoliosis, equinovarus foot deformity, joint contractures.
Non-ortho manifestations: cardiomyopathy, static encephalopathy
Progressive weakness affecting proximal muscles first (begins with gluteal muscle weakness), gait abnormalities, delayed walking, toe walking, clumsy, waddling gait, difficulty climbing stairs, hopping, jumping, decreased motor skills.
Present deep tendon reflexes (unlike SMA), lumbar lordosis (to compensate for gluteal weakness), Gower’s sign (rises by walking hands up legs to compensate for gluteus maximus and quads weakness), trendelenberg sign.
Muscle biopsy - absent dystrophin, connective tissue infiltration and foci of necrosis.
CK leaks across defective cell membrane.
DNA testing - absent dystrophin.
EMG_ myopathic - decreased amplitude, short duration, polyphasic motor.

Differentiation DMD vs SMA: similarities are proximal weakness, but SMA onset earlier in children, deep tendon reflexes and fasciculations are absent. Normal CK levels. No pseudo hypertrophy

Differentiation DMD vs Emery-Dreifuss Dystrophy: similar clinical picture but no calf pseudo hypertrophy, Near normal CK levels, early development of elbow and ankle contractures.

Friedreichs Ataxia: inherited disorder leading to spinocerebellar degeneration
AR mutation in Frataxin gene.
Presentation: ataxia, cardiomyopathy, motor weakness, cavovarus foot, scoliosis
Dx: genetic testing - absence of frataxin gene (mitochondrial protein GAA repeat at 9q13)
Classic triad - ataxia, areflexia, positive plantar response.
Nystagmus, cavovarus foot (high arch, rigid deformity, claw toes), scoliosis.
Usually wheelchair bound by 30, die from cardiomyopathy by 50.

Charcot Marie Tooth - peroneal muscular atrophy AD hereditary motor sensory neuropathy. PMP22 (expressed in Schwann cells) Chr 17 - may also be AR or X-linked.
Abnormal peripheral myelin protein - presents with muscle weakness and sensory changes that lead to cavovarus feet, scoliosis and claw foot deformity.
Dx: NCS - low velocities , prolonged distal latencies in perineal, ulnar and median nerves.
Most common inherited progressive peripheral neuropathy.
HMSN Type I: abnormal myelin sheath protein -> combo motor and sensory disturbances.
HMSN Type II: ntact myelin sheath with wallerian axonal degeneration that results in mild sensory and motor conduction velocities.
Weakness :
- profound peroneus brevis first (leading to imbalance and varus deformity)
- tib ant (leading to foot drop)
- intrinsic muscles hand and foot (wasting 1st dorsal interosseous in hands)
- Unopposed pull of peroneus longus and tib post -> plantar flexion first ray and compensatory hind foot varus, initially flexible then becomes rigid.
Coleman block test: to determine if hind foot varus is secondary to plantar flexed 1st ray vs independent component - if corrects with coleman block then varus is forefoot driven, if does not correct suggest hind foot driven varus deformity. Rigid hind foot will not correct to neutral.

60
Q
  1. You review a 19 year old male patient in outpatients who suffered a progressive muscle weakness and has been declining recently. He has a waddling gait and a positive Gower’s sign. He walks on tip toes and you note calf enlargement. Which is best to confirm your diagnosis?

a. Genetic testing
b. EMG
c. Muscle biopsy and immunohistochemistry
d. Creatine Kinase
e. MRI calf

A

Answer: C

DMD = dystrophin absence, young males only, dystrophin absence leads to poor muscle fibre regeneration and progressive replacement of muscle tissue with fibrous and fatty tissue, therefore skeletal and cardiac muscle lose elasticity and strength.
X-linked recessive
Xp21.2 dystrophin gene defect (point deletion, third spontaneous mutations)
Ortho manifestations: calf pseudo hypertrophy, scoliosis, equinovarus foot deformity, joint contractures.
Non-ortho manifestations: cardiomyopathy, static encephalopathy
Progressive weakness affecting proximal muscles first (begins with gluteal muscle weakness), gait abnormalities, delayed walking, toe walking, clumsy, waddling gait, difficulty climbing stairs, hopping, jumping, decreased motor skills.
Present deep tendon reflexes (unlike SMA), lumbar lordosis (to compensate for gluteal weakness), Gower’s sign (rises by walking hands up legs to compensate for gluteus maximus and quads weakness), trendelenberg sign.
Muscle biopsy - absent dystrophin, connective tissue infiltration and foci of necrosis.
CK leaks across defective cell membrane.
DNA testing - absent dystrophin.
EMG_ myopathic - decreased amplitude, short duration, polyphasic motor.

Differentiation DMD vs SMA: similarities are proximal weakness, but SMA onset earlier in children, deep tendon reflexes and fasciculations are absent. Normal CK levels. No pseudo hypertrophy

Differentiation DMD vs Emery-Dreifuss Dystrophy: similar clinical picture but no calf pseudo hypertrophy, Near normal CK levels, early development of elbow and ankle contractures.

Friedreichs Ataxia: inherited disorder leading to spinocerebellar degeneration
AR mutation in Frataxin gene.
Presentation: ataxia, cardiomyopathy, motor weakness, cavovarus foot, scoliosis
Dx: genetic testing - absence of frataxin gene (mitochondrial protein GAA repeat at 9q13)
Classic triad - ataxia, areflexia, positive plantar response.
Nystagmus, cavovarus foot (high arch, rigid deformity, claw toes), scoliosis.
Usually wheelchair bound by 30, die from cardiomyopathy by 50.

Charcot Marie Tooth - peroneal muscular atrophy AD hereditary motor sensory neuropathy. PMP22 (expressed in Schwann cells) Chr 17 - may also be AR or X-linked.
Abnormal peripheral myelin protein - presents with muscle weakness and sensory changes that lead to cavovarus feet, scoliosis and claw foot deformity.
Dx: NCS - low velocities , prolonged distal latencies in perineal, ulnar and median nerves.
Most common inherited progressive peripheral neuropathy.
HMSN Type I: abnormal myelin sheath protein -> combo motor and sensory disturbances.
HMSN Type II: ntact myelin sheath with wallerian axonal degeneration that results in mild sensory and motor conduction velocities.
Weakness :
- profound peroneus brevis first (leading to imbalance and varus deformity)
- tib ant (leading to foot drop)
- intrinsic muscles hand and foot (wasting 1st dorsal interosseous in hands)
- Unopposed pull of peroneus longus and tib post -> plantar flexion first ray and compensatory hind foot varus, initially flexible then becomes rigid.
Coleman block test: to determine if hind foot varus is secondary to plantar flexed 1st ray vs independent component - if corrects with coleman block then varus is forefoot driven, if does not correct suggest hind foot driven varus deformity. Rigid hind foot will not correct to neutral.

61
Q
  1. A 41 year old male sustained an inversion injury to ankle following a football tackle. He is still in considerable amount of pain 8 weeks after the injury. There was minimal tenderness over the deltoid ligament and ankle instability tests were normal. What is the most likely injury picked up on an MRI scan that requires further treatment?

A. Fracture of 5th metatarsal base
B. Osteochondral injury to talus
C. Fracture of lateral malleolus
D. Fracture of medial malleolus
E. Deltoid ligament injury

A

Answer: B

Osteochondral Lesions of the Talus are focal injuries to the talar dome with variable involvement of the subchondral bone and cartilage which may be caused by a traumatic event or repetitive microtrauma.
Diagnosis can be made with plain ankle radiographs. MRI studies are helpful in determining the size of the lesion, the extent of bony edema, and identify unstable lesions.
Treatment can be nonoperative or operative depending on patient age, patient activity demands, lesion size, and stability of lesion.

mechanism of injury
ankle inversion and dorsiflexion during axial load creates shearing of lateral talar dome and lateral OLT
ankle inversion, external rotation, and plantarflexion during axial load creates shearing of medial talar dome and medial OLT
pathophysiology
possible repeitive microtrauma creates ischemic environment and loss of integrity of subchondral bone
leads to softening and disruption of overlying cartilage
Associated conditions
cavus hindfoot alignment

Relies on extra-osseous blood supply: deltoid artery supplies majority of talar body and dome

Typical history: inversion ankle sprain
Causes pain centred over ankle joint, joint effusion, mechanical; symptoms (catching or locking). Motion is often limited due to pain or effusion.

XRs: often normal, sometimes subtle lucency or bone fragmentation.

Non-op: acute injury, non-displaced fragment with incomplete fracture.

Arthroscopy, removal fragment, debridement & marrow stimulation: chronic lesions, size <1cm, displaced small fragment with minimal bone on the osteochondral fragement (poor healing potential)

Retrograde drilling and/or bone grafting: size >1cm with intact cartilage cap

Osteochondral grafting (autograft transplant, autologous chondrocyte implantation, bulk allograft): size >1cm, displaced lesions, shoulder lesions, salvage for failed marrow stimulation or drilling. CI in diffuse ankle arthritis, bipolar kissing lesions, advanced osteonecrosis of taller dome.

62
Q
  1. A 25 year old professional golfer sustained a severe ankle sprain following an inversion injury. XR immediately following the injury showed a fleck sign over the lateral malleolus. He reports ongoing symptoms of instability after 6 weeks treatment in a short leg cast.
    What is the most likely treatment option which will result in a quicker return to golf?

a. Debridement of peroneal tendons and FHL transfer
b. Tenodesis of peroneus brevis tendon to peroneus longus tendon
c. Repair of lateral ligament complex
d. Repair of superficial peroneal retinaculum +/- deepening of retromalleolar groove
e. Ankle arthroscopy and proceed

A

Answer: D

Peroneal Tendon Tears and Instability represent a spectrum of traumatic injuries to the lateral ankle that include tenosynovitis, tendinopathy, tendon tears and/or tendon instability.
Diagnosis is made clinically with subfibular ankle pain with the sensation of apprehension or subluxation with active dorsiflexion and eversion against resistance. MRI studies can help identify the size of peroneal tendon tear and identify concomitant injuries to nearby structures.
Treatment may be nonoperative or operative depending on patient activity demands, chronicity of injury, and peroneal instability.

mechanism of injury
rapid forced dorsiflexion of the inverted foot will cause strain through the contracted peroneal muscles, leading to superior peroneal retinaculum (SPR) tear - (most common pattern is longitudinal split tear in the PB)
If superior peroneal retinaculum tears, tendons will become unstable and subluxate or completely dislocate.

Peroneus brevis (PB)
innervated by the superficial peroneal nerve, S1
acts as primary evertor of the foot
tendinous about 2-4cm proximal to the tip of the fibula
lies anterior and medial to the peroneus longus at the level of the lateral malleolus

Peroneus longus (PL)
innervated by superficial peroneal nerve, S1 primarily a plantar flexor and foot and first metatarsal
can have an ossicle (os peroneum) located within the tendon body near the calcaneocuboid joint

Peroneal tendons contained within a common synovial sheath that splits at the level of the peroneal tubercle.

The sheath runs in the retromalleolar groove on the fibula. Peroneus brevis is directly posterior to the fibula at the level of the groove. Peroneus longus is directly posterior to peroneus brevis at the level of the groove, which is deepened by a fibrocartilaginous rim (still only about 5 millimeters deep) and covered by superior peroneal retinaculum (SPR).
SPR originates from the posterolateral ridge of the fibula and inserts onto the lateral calcaneus (peroneal tubercle). The inferior aspect of the SPR blends with the inferior peroneal retinaculum and is the primary restraint of the peroneal tendons within the retromalleolar sulcus

At the level of the peroneal tubercle of the calcaneus: peroneus longus is inferior & peroneus brevis is superior

Both tendons covered by inferior peroneal retinaculum

Blood supply: supplied by branches of the anterior and posterior tibial arteries via vincula system - entirety of both tendons are vascularized

History: feeling a pop with a distinct dorsiflexion ankle injury, feelings of instability in lateral ankle, lateral or posterolateral ankle pain. Swelling posterior to lateral malleolus, cavovarus hind foot alignment, voluntary subluxation of tendons and a ‘popping’ sound.

XR: “Fleck sign’ cortical avulsion of the SPR off the distal tip of the lateral malleolus.
Plantarflexed first metatarsal and high Meary’s angle, indicating cavovarus deformity. Proximal migration of the os peroneum is indicative of peroneus longus rupture.

Non-op: all acute PB/PL instability in non-profession athletes.

Repair of SPR and deepening retromalleolar groove: acute tendon dislocations in high level athletes who desire a quick return to sport/presence of a longitudinal tear.

Groove deepening with soft tissue transfer and/or osteotomy: chronic dislocations with bony abnormalities or incompetent SPR - generally a salvage procedure.

Tenosynovectomy and tendon debridement with tubularization: recalcitrant and symptomatic PB/PL tears less than 50-60% of the tendon width

Tenodesis of distal and proximal ends of the brevis tendon to the peroneus longus: complex tears of the brevis tendon with multiple longitudinal tears and significant tendinosis (> 50% of the tendon involved)

Debridement of both tendons with FHL/FDL transfer: complex tears of both tendons (involving over 50% of tendon substance) with no muscle excursion

63
Q
  1. During embryological limb bud development, radial-ulna growth is co-ordinated through which mechanism?

a. FGF expression in the Zone of Polarising Activity
b. WNT7a signalling in the Apical Ectodermal Ridge
c. WNT7a expression in the dorsal ectoderm
d. SHH expression in the Zone of Polarising Activity
e. FGF expression in the Apical Ectodermal Ridge

A

Answer: D

Sonic Hedgehog (Shh) genes
secreted by ZPA, involved with HOX gene expression
- anterior-posterior: (radioulnar) growth: anterior (radial) mesoderm expresses ALX4, posterior (ulnar) mesoderm expresses Hox8

the appendicular system forms between the 4-8 weeks of gestation

limb bud development
- appears to be under the control of fibroblast growth factors (FGF)
- enlargement of the limb bud is due to the interaction between the apical ectodermal ridge (AER) and the mesodermal cells in the progress zone.
- first identifiable by transvaginal ultrasound at 8 weeks

Steps of limb development:
- notochord expresses Shh
- Shh regulates limb bud formation: limb bud is combination of lateral plate mesoderm and somatic mesoderm growing outwards into ectoderm (called apical ectodermal ridge). Limb bud formed at embryonic stage 12 (26 days after fertilization)

  • Mesenchyme condenses into preskeletal blastemal at core of limb bud
  • Chondrification occurs where mesenchyme differentiates into chondrocytes. All upper limb bones are endochondral except distal parts of distal phalanges (membranous). From proximal (humerus, 36 days after fertilization) to distal (distal phalanges, 50 days)
    Factors required for chondrification: transcription factors – Sox-5, Sox-6, Sox-9
    transforming growth factor superfamily – TGF-b, BMP-2
    FGF family (receptor mutation leads to acrocephalosyndactyly (Apert syndrome), retinoids, hedgehog gene products, PTHrP, cadherins, WNT5a and WNT7a
  • Formation of joints requires repression of chondrogenesis at sites of future joints
    proteins involved – WNT4, WNT14, growth and differentiation factor 5 (also known as cartilage-derived morphogenetic protein 1)
    shoulder interzone appears at 36 days, hand interzones appear at 47 days

Limb patterning
Proximodistal : first signaling center to appear is AER
controls proximal to distal growth. forms under FGF10 stimulation
Removal /defect in AER results in proximal limb truncation - example is central deficiency (cleft hand), another example is radial clubhand (radial dysplasia, absence of radius)
FGFs are the signaling molecule. FGFs expressed in AER include FGF4, FGF8, FGF9, and FGF 17
FGF8 expressed earliest and is obligatory for normal limb development
FGF4, 9 and 17 are redundant
disrupted FGF signalling leads to arrested limb development

Anteroposterior
second signaling center to appear is ZPA (zone of polarizing activity), along posterior limb bud
controls anteroposterior (radioulnar) limb growth
signaling molecule is Shh compound (dose dependent)
- normal: high concentration of Shh on posterior (ulnar) side for small finger development and low concentration of Shh on anterior (radial) side for thumb development

posterior/ulnar side abnormalities
- abnormal upregulation of Shh in the ZPA results in polydactly on the ulnar (posterior) side- extent of duplication is dose dependent (higher dose = more replication)

  • downregulation of Shh (on the posterior/ulnar side) leads to loss of ulnar digits
    anterior/radial side abnormalities
  • abnormal upregulation of Shh in the anterior aspect of the limb bud (where Shh concentration is supposed to be low) leads to loss of thumb

timing
posterior elements (little finger/ulna) are formed EARLY prior to anterior elements which are formed LATE (radius/thumb)
disruption of AP patterning will result in loss of later forming elements (radius/thumb)

Dorsoventral axis
- third signaling center is non-AER limb ectoderm /Wnt signalling center (progress zone, PZ)
- dorsal limb ectoderm expresses WNT7a
activates Lmx1b (LIM-homeodomain factor) to regulate dorsal patterning
WNT7a is responsible for all dorsal features (including nails)
ventral ectoderm expresses en-1 (engrailed-1 protein, antagonistic to WNT7a)
inhibits WNT7a (and restricts it to dorsal ectoderm)
allows ventral limb development

64
Q
  1. A 14 year old boy presents with leg pain of 2 months. Radiograph’s show onion peel appearance with histology showing uniform small round blue cells. Choose the genetic defect association with this condition.

A. Mutation retinoblastoma protein (pRB-1) coding gene
B. Mutation in epidermal growth factor protein (erbB-2) coding gene
C. Translocation t(11:22)
D. Translocation t(12:22)
E. Mutation in tumour protein 53 (p53) coding gene

A

Answer: C

Ewing’s Sarcoma is a malignant, distinctive small round cell sarcoma associated with a t(11:22) translocation which most commonly occurs in the diaphysis of long bones in patients <25 with regional pain, swelling and fevers.
Diagnosis is made with a biopsy showing sheets of monotonous small round blue cells with prominent nuclei and minimal cytoplasm and immunostaining positive for CD99.
Treatment is usually neo-adjuvant chemotherapy and limb salvage surgical resection, followed by adjuvant chemotherapy +/- radiation.

t(11:22) translocation found in 85-95% of cases leads to the formation of a fusion protein (EWS-FLI1)

XR: large distractive lesion in diaphysis or metaphysics with ill-defined, permeative, moth-eaten appearance. Lytic lesion often with variable amounts of new bone formation.
Periosteal reaction may give onion skin or sunburst appearance. Large associated soft tissue mass in >80% cases.
Elevated ESR, WCC & LDH
Histology: monotonous small round blue cells, high nuclei: cytoplasm ratio. May have pseudo-rosettes (circles of cells with necrosis in centre).
Immunistaining: CD99 (+ve in 95%),

Standard of care:
- Chemo (neoadjuvant (8-12 weeks) + adjuvant (6-12 months))
-> surgery - goal is to obtain local control and prevent late recurrence of chemoresistant cells. Wide margins -> improved 5 year survival.
+/- adjuvant radioTx - not necessary if margins are adequate and there is a good response to chemo. Indicated if positive post- surgical resection margins or presence of pulmonary mets.

65
Q

. A 75-year-old man is admitted with septic arthritis of a THR. He is septic and at the time of surgery his hip is frankly infected. It is decided to perform a resection arthroplasty. His family ask you what the chances of recurrence of infection in his new hip are:

a. Less than 10%
b. 10-20%
c. 30-40%
d. 40-50%
e. 50-60%

A

Answer: B

Incidence
- Primary joint replacement: 1-2% TKA vs. 0.3-1.3% THA
- Revision joint replacement: 5-6% TKA vs. 3-4% THA

Risk factors:
- Immunosuppressant drugs or conditions
- Inflammatory arthropathy
Lifestyle: morbid obesity, smoking, EtOH, IVDU, poor oral hygiene

most common bacterial organism include
staphylococcus aureus
staphylococcus epidermidis
Coagulase-negative Staphylococcus (chronic infections)

most common fungal pathogen
Candida species (e.g. Candida albicans)

Major criteria (diagnosis can be made when 1 major criteria exist)
sinus tract communicating with prosthesis
pathogen isolated by culture from 2 separate tissue/fluid samples from the affected joint
Minor criteria (preoperative diagnosis)
The below scores are added together to determine:
≥6 = infected; 2-5= inconclusive; 0-1=not infected
Serum Labs
Elevated CRP (>10mg/L) or D-dimer (>860ng/mL) - 2 points
Elevated ESR (>30mm/h) - 1 point
Synovial Fluid Analysis
Elevated synovial WBC (>3,000 cells/µl) or Leukocyte Esterase - 3 points
Positive alpha-defensin - 3 points
most sensitive and specific marker for PJI
Elevated synovial PMN (>80%) - 2 points
Elevated synovial CRP (>6.9mg/L) - 1 point
Inconclusive (inconclusive preop score (2-5) or dry aspiration)
Positive histology (>5 PMN per hpf in 5 hpf at x400 magnification (intraoperative frozen section of periprostehtic tissue) - 3 points
Purulence in affected joint - 3 points
Single positive culture - 2 points
Preoperative score + intraoperative score combined
Combined score ≥6 = infected; 4-5= inconclusive; 0-3= not infected

Nonoperative
chronic suppressive antibiotic therapy
indications
unfit for surgery
refuse surgery
systemic spread and maintain joint motion with symptomatic relief
outcomes
10% to 25% success rate of eradication
8% to 21% complication rate

polyethylene exchange with component retention, IV abx for 4-6 weeks
indications
acute infection (<3 weeks after surgery)
acute hematogenous infection (weak literature, ideally <48-72hrs from symptom onset)
techniques
thorough tissue debridement and irrigation with large-volume of irrigant
outcomes
50% to 55% success rate
implants must be removed if reinfection documented
Dependant of bacteria speciation

one-stage replacement arthroplasty
indications
used more commonly in Europe for infected THA
no sinus tract, healthy patient and soft tissue, no prolonged antibiotic use, no bone graft
low-virulence organism with good antibiotic sensitivity
technique
use antibiotic-impregnated cement
advantages
lower cost and convenience with single procedure
earlier mobility
disadvantages
higher risk of continued infection from residual microorganisms
outcomes
variable success of 75-100%

two-stage replacement arthroplasty
indications
gold standard for an infected joint >4 weeks after arthroplasty
must be medically fit for multiple surgeries
requires adequate bone stock
requires confirmation of microbial eradication
benign clinical exam
normal labs (WBC, ESR, and CRP)
negative aspiration cultures
obtain repeat cultures at least two weeks after planned antibiotic course has been completed
techniques (see section below)
prosthesis removal, antibiotic spacer, IV antibiotics for 4-6 weeks and delayed reconstruction
outcomes
bilateral TKA resection arthroplasty followed by 6 weeks of antibiotics and bilateral reimplantation has excellent results at 2-year follow-up
early reimplantation within 2 weeks has 35% success rate
delayed reimplantation >6 weeks has a 70-90% success rate
cementless reimplantation in the hip has better outcomes than cemented

resection arthroplasty
indications
poor bone and soft tissue quality
recurrent infections with multi-drug resistant organisms
medically unfit for multiple surgeries
failure of multiple previous reimplantations
elderly nonambulatory patients
disadvantages
short limb, poor function, and patient dissatisfaction
technique
remove all infected tissue and components with no subsequent reimplantation
outcomes
total knee success rate is 50% to 89%
total hip success rate is 60% to 100%

arthrodesis
indications
reimplantation is not feasible due to poor bone stock
recurrent infections with virulent organisms
outcomes
71% to 95% success rate with bony fusion and infection eradication

Amputation
total knee infections recalcitrant to other options
severe pain, soft tissue compromise, severe bone loss, or vascular damaged
AKA

66
Q
  1. Which of the following arthrodesis is most likely to benefit a patient with stage III SLAC wrist?

a. STT fusion with radial styloid excision
b. Four corner fusion with scaphoid excision
c. Four corner fusion without scaphoid excision
d. Scapho-capitate fusion with radial styloid excision
e. Four corner fusion with radial styloid excision

A

Answer: B

Scaphoid Lunate Advanced Collapse (SLAC) describes the specific pattern of degenerative arthritis seen in chronic dissociation between the scaphoid and lunate.

chronic SL ligament injury creates a DISI deformity
scaphoid is flexed and lunate is extended as scapholunate ligament no longer restrains this articulation
scapholunate angle > 70 degrees
lunate extended > 10 degrees past neutral
resultant scaphoid flexion and lunate extension creates
abnormal distribution of forces across midcarpal and radiocarpal joints
malalignment of concentric joint surfaces
initially affects the radioscaphoid joint and progresses to capitolunate joint
notably the radiolunate joint is spared

Watson Classification:
I: Arthritis between scaphoid and radial styloid -> Radial styloidectomy // PIN, AIN denervation (can be combined with any treatments for II or III)
II: Arthritis between scaphoid and entire scaphoid facet of the radius –> proximal row carpectomy (CI if caputolunate arthritis or incompetent radioscapholunate ligament) then scaphoid excision & 4-corner fusion
III: Arthritis between capitate and lunate –> 4-corner fusion or wrist fusion (any form of pan carpal arthritis).
N.B. Radiolunate joint is spared.
Theoretical Type IV in which there is pan carpal arthritis with radiolunate joint affected.

67
Q
  1. Which of the following statements about trigger thumb is incorrect?

a. Surgical intervention is often required to treat this condition
b. Release of the A1 pulley is not usually sufficient to treat this condition
c. The radial digital nerve is more commonly injured than the ulnar digital nerve
d. There have been similar reports of safety and efficacy between open and percutaneous release techniques
e. Trigger thumb release does not have a high recurrence rate

A

Answer: B
Percutaneous release of A1 pulley has >90% success rate

Trigger Finger (trigger thumb when involving the thumb) is the inhibition of smooth tendon gliding due to mechanical impingement at the level of the A1 pulley that causes progressive pain, clicking, catching, and locking of the digit.
Diagnosis is made by physical examination with presence of active triggering and tenderness at the A1 pulley.
Treatment consists of splinting, anti-inflammatory medications, steroid injections, and surgical release.
Risk factor: diabetes.
Associated with carpal tunnel syndrome (>60% patients), RA, calcific tendinitis, septic tenosynovitis, amyloidosis, hypothyroidism, sarcoidosis, gout, pseudo gout
Ring and long fingers most commonly involved in adults.

Occurs due to stenosing tenosynovitis at the A1 pulley. Fibrocartilaginous metaplasia of tendon and/or pulley, proliferation chondrocytes, increased type III collagen, chronic hyperglycaemia creates collagen cross-links and impairs collagen degradation.
Occasional pathologic nodule of FDP tendon, FDS often unaffected, trigger thumb may have a 4th pulley (variable annular pulley) causing stenosis in up to 75% patients.

release of A1 pulley and 1 slip of FDS (usually ulnar slip)
indications: persistent/recurrent triggering after A1 pulley release
rheumatoid arthritis patients may benefit from FDS slip excision without A1 pulley release
pediatric trigger finger
presents with Notta’s node (proximal to A1 pulley), flexion contracture and triggering
surgical treatment at 2-4 years of age to prevent interphalangeal joint contracture
may need to release remaining FDS slip and A3 pulley

Radial digital nerve injury
risk factors
- trigger thumb release due to superficial location and oblique orientation
treatment: may require digital nerve and artery repair

68
Q
  1. A patient presents to your clinic with aseptic loosening of a total knee replacement. Polyethylene wear debris causes synovial activation and release of inflammatory mediators in the knee.

Which cell type or mediator repairs damaged synovial membrane?

A. Type A synovial macrophages
B. Prostaglandin (PGE2)
C. Type B synovial fibroblasts
D. Tumour necrosis factor
E. Type A osteoclasts

A

Answer: C

Synovium mediates nutrient exchange between blood and joint fluid - made from a ultrafiltrate of blood plasma

synovial fluid exhibits non-Newtonian flow characteristics - the viscosity coefficient is not a constant. The fluid is not linearly viscous
viscosity increases as the shear rate decreases

Composition
vascularized connective tissue is porous and lacks basement membrane
cell types

type A cells: derived from macrophages, non-fixed cells with antigen presenting ability located in superficial layer and important in phagocytosis

type B cells
fibroblast like cells
rich rough endoplasmic reticulum and dendritic processes that reach out to the joint surface
located at various depths, frequently in deeper layer
produce synovial fluid
(produce hyaluronic acid, fibronectin, collagen)

type C cells
intermediate cell type
unknown function and origin
may serve as multi-potent precursor to either type A or B synovial cells

69
Q

. Which is NOT a risk factor for dislocation of a total hip replacement?

A. Social Deprivation
B. Drug Use
C. Gender
D. Obesity
E. Avascular Necrosis

A

Answer: A

Female sex, obesity, and neurological conditions represent patient-related risk factors for dislocation.4-8 Small-sized femoral head components, (28 mm or less) are known to have a higher risk of dislocation compared to THAs using larger femoral heads (32 mm or greater).

Incidence 1-3%
70% occur within first month
75-90% posterior

Mechanism
- anterior: extension and external rotation of hip
- posterior: flexion, internal rotation, adduction of hip

Risk factors
- Prior hip surgery (greatest risk factor)
- obesity
>70-80 years of age
posterior surgical approach
repairing capsule and reconstructing external rotators brings dislocation rate close to anterior approach
malpositioning of components
ideal positioning of acetabular component is 40 degrees of abduction and 15 degrees anteversion
in general, excessive anteversion increases risk of anterior hip dislocation; excessive retroversion increases risk of posterior hip dislocation
spastic or neuromuscular disease (Parkinson’s)
drug or alcohol abuse
decreased femoral offset (decreases tissue tension and stability)
decreased femoral head to neck ratio
prior spinal fusion or fixed spinopelvic alignment
polyethylene wear
common cause of late instability occuring >5 years after procedure

Dislocation is one of the most common causes of patient and surgeon dissatisfaction following hip replacement and to treat it, the causes must first be understood. Patient factors include age greater than 70 years, medical comorbidities, female gender, ligamentous laxity, revision surgery, issues with the abductors, and patient education. Surgeon factors include the annual quantity of procedures and experience, the surgical approach, adequate restoration of femoral offset and leg length, component position, and soft-tissue or bony impingement. Implant factors include the design of the head and neck region, and so-called skirts on longer neck lengths. There should be offset choices available in order to restore soft-tissue tension. Lipped liners aid in gaining stability, yet if improperly placed may result in impingement and dislocation. Late dislocation may result from polyethylene wear, soft-tissue destruction, trochanteric or abductor disruption and weakness, or infection. Understanding the causes of hip dislocation facilitates prevention in a majority of instances. Proper pre-operative planning includes the identification of patients with a high offset in whom inadequate restoration of offset will reduce soft-tissue tension and abductor efficiency. Component position must be accurate to achieve stability without impingement. Finally, patient education cannot be over-emphasised, as most dislocations occur early, and are preventable with proper instructions.

Cite this article: Bone Joint J 2013;95-B, Supple A:67–9.

A history of spinal fusion was the strongest independent predictor of dislocation (odds ratio [OR], 2.45; 95% confidence interval [CI], 1.97-3.04; P < .0001). Parkinson’s disease was also significantly associated with dislocation (OR, 1.63; 95% CI, 1.05-2.51; P = .03), as well as dementia (OR, 1.96; 95% CI, 1.13-3.39; P = .02), depression (OR, 1.28; 95% CI, 1.13-1.43; P < .0001), and chronic lung disease (OR, 1.2; 95% CI, 1.07-1.33; P = .001). Inflammatory arthritis and avascular necrosis were independent risk factors for dislocation (OR, 1.56; 95% CI, 1.25-1.97; P < .0001; OR, 1.67; 95% CI, 1.45-1.93; P < .0001).

70
Q

A 30 year old male patient presents to the emergency department with a hip fracture after falling down a flight of stairs.

 needs XR images!!!! XR spine: "rugger jersey spine"

The following are true with regard to the patient’s underlying diagnosis, EXCEPT:

A. These patients may give a history of progressive deafness
B. These patients may give a history of progressive visual problems
C. Anaemia is a recognised feature of this condition
D. X-rays of the distal femur frequetly demonstrate Erlenmeyer flask deformity
E. This condition exhibits x-linked recessive genetics

A

Answer: E - Osteopetrosis is either AD (1/20,000) or AR (1/200,000)

Benign AD is commonest form - usually asymptomatic

AR forms - frequent fractures, progressive deafness and blindness, severe anaemia, bleeding risk, frequent infections. OM of mandible, dental abscesses, hepatosplenomegaly.

Osteopetrosis is a congenital metabolic bone disease caused by defective osteoclastic resorption of immature bone that presents with increased frequency of long bone fractures, cranial nerve palsies, and low back pain.
Diagnosis is made radiographically with increased cortical thickening, increased overall bone density, and loss of medullary canal diameter.
Treatment is multidisciplinary approach to address fracture management, and cranial nerve abnormalities.

Caused by osteoclast inability to acidify Howships lacuna (defective carbonic anhydrase II or chloride channel dysfunction) - leads to predisposition to fracture (lower»upper»axial skeleton)

lack ruffled border and clear zone, islands of calcified cartilage with mature trabeculae

Associated with:
- CN palsies (overgrowth of skull foramina) optic n> auditory n> trigeminal n> facial n.
- osteomyelitis (lack of marrow vascularity -> impaired WBC function)
- Increased prevalence spondylolysis
- Coxa vara - often due to fem neck fracture non-union or repeated stress fractures.
- Carpal tunnel syndrome

71
Q
  1. A 41 year old woman presents with a constant left shoulder pain that is becoming more severe. Imaging reveals the lesion shown below. Needle biopsy reveals a high grade cartilaginous tumour with more than 80% necrosis of the tissue. Staging reveals no metastases. Which is the most appropriate management?

a. Clinical re-examination and imaging in 6 months
b. Curettage and bone graft
c. Curettage with argon bean ablation of the cavity
d. Curettage with argon beam ablation of the cavity, calcium sulfate and ORIF
e. Wide local excision and proximal humeral replacement

A

Answer: E

72
Q
  1. Based on the following lactate measures, in which case would damage control orthopaedics be the most appropriate?

a. Initial lactate of 4 with current lactate of 2
b. Initial lactate of 1.5 with a current lactate of 1.5
c. Initial lactate of 2 with current lactate of 4
d. Initial lactate of 6 with current lactate of 2.5
e. A single reading of 4

A

Answer: C

Damage control orthopaedics: definitive treatment delayed until physiology has improved

Parameters that help decide who should be treated with DCO
ISS >40 (without thoracic trauma)
ISS >20 with thoracic trauma
GCS of 8 or below
lactate >4

multiple injuries with severe pelvic/abdominal trauma and hemorrhagic shock
bilateral femoral fractures
pulmonary contusion noted on radiographs
hypothermia <35 degrees C
head injury with AIS of 3 or greater
IL-6 values above 500pg/dL

Optimal time of surgery
patients are at increased risk of ARDS and multisystem failure during the acute inflammatory window (period from 2 to 5 days characterized by a surge in inflammatory markers)
therefore only potentially life-threatening injuries should be treated in this period including
unstable pelvic fracture
compartment syndrome
fractures with vascular injuries
unreduced dislocations
traumatic amputations
unstable spine fractures
cauda equina syndrome
open fractures

Trauma is a major public health problem with high disability, death, and societal cost
Three peak times of death after trauma
50% within the first minutes of sustaining the injury
caused by massive blood loss or neurologic injury
30% within hours of arrival to hospital
most commonly from shock, hypoxia, or neurologic injury
20% within days to weeks following injury
multi system organ failure and infection are leading causes
Golden Hour
period of time when life threating and limb threatening injuries should be treated in order to decrease mortality
estimated 60% of preventable deaths can occur during this time ranging from minutes to hours

73
Q
  1. When considering patients with a calcaneal fracture, which of the following is most likely:

a. An increased Bohler’s angle
b. Damage to the saphenous nerve if fixed
c. 50% risk of ipsilateral injury
d. 20% risk of lumbar spine fracture
e. ‘Blow out’ of the medial wall of the calcaneum

A

Answer: D

XR: double density sign (represents subtalar incongruity), calcaneal shortening, varus tuberosity deformity, decreased Bohlers angle (initial Böhler’s angle <0° (these injuries do poorly regardless of treatment)
lower Böhler angles suggest greater energy absorbed), increased angle of Gissane

Calcaneus fractures are the most common fractured tarsal bone and are associated with a high degree of morbidity and disability.
Diagnosis is made radiographically with foot radiographs with CT scan often being required for surgical planning.
Treatment is nonoperative versus operative based on fracture displacement and alignment, associated soft tissue injury, and patient risk factors.
17% are open fractures

intra-articular fractures
traumatic axial loading is the primary mechanism of injury
fall from height
motor-vehicle accidents

calcaneal tuberosity fractures
poor bone quality/osteoporosis
violent contaction of the triceps surae with forced dorsiflexion
strong concentric contaction of the triceps surae with knee in full extension
intrinsic tightness of the gastrocnemius and achilles tendon
peripheral neuropathy leading to decreased pain sensation and proprioception resulting in recurrent microtrauma

calcaneal stress fractures
increased physical activity in the setting of relative energy deficiency

anterior process fractures

twisting injury mechanism
avulsion injury of the bifurcate ligament

intra-articular fractures
primary fracture line results from oblique shear and leads to the following two primary fragments
superomedial fragment (constant fragment)
includes the sustentaculum tali and is stabilized by strong ligamentous and capsular attachments
superolateral fragment
includes an intra-articular aspect through the posterior facet
secondary fracture lines
dictate whether there is joint depression or tongue-type fracture
Intra-articular (75%)
Essex-Lopresti classification
the primary fracture line runs obliquely through the posterior facet forming two fragments
the secondary fracture line runs in one of two planes
the axial plane beneath the facet exiting posteriorly in tongue-type fractures
when the superolateral fragment and posterior facet remain attached to the tuberosity posteriorly
behind the posterior facet in joint depression fractures
Sanders classification
based on the number of articular fragments seen on the coronal CT image at the widest point of the posterior facet

extra-articular fractures
strong contraction of gastrocnemius-soleus with concomitant avulsion at its insertion site on calcaneus
more common in osteopenic/osteoporotic bone.
Extra-articular (25%)
avulsion injury of
anterior process by bifurcate ligament
sustentaculum tali
calcaneal tuberosity (Achilles tendon avulsion)

Surgical management has high risk of wound complications (extensile lateral L-shaped incision is commonest). Lateral calcaneal branch of peroneal artery supplies the full thickness skin, soft tissue and periostea flaps that are developed.

Complications: (40% rate)
- wound complication
- subtalar arthritis
- lateral impingement with peroneal irritation
- sural nerve neuroma
- Damaged FHL
- compartment syndrome
- malunion

74
Q
  1. Which of the following extrinsic wrist ligaments is NOT volar?

A. Short radiolunate ligament
B. Long radiolunate ligament
C. Radiotriquetral ligament
D. Radioscapholunate ligament
E. Ulnotriquetral

A

Answer: C

75
Q
  1. An 82-year-old woman falls from a standing height and sustains a proximal humerus fracture. Which factor is the best predictor of ischemia of the humeral head?

A. Fracture pattern involving 4 parts
B. Humeral head angulation exceeding 45 degrees
C. Posteromedial calcar length of less than 8 mm attached to the humeral head
D. Glenohumeral dislocation
E. Two part fracture pattern

A

Answer: C

76
Q
  1. The Smith-Peterson approach uses the interval between muscles supplied by which 2 nerves?

a. Femoral and sciatic nerve
b. Femoral and inferior gluteal nerve
c. Femoral and superior gluteal nerve
d. Superior gluteal and obturator nerves
e. Superior gluteal and sciatic nerves

A

Answer: C

77
Q
  1. All the following statements are true, EXCEPT:

a. Talar neck fractures are high energy injuries, usually resulting from forced dorsiflexion of the foot
b. The dominant supply to the talar body is via the artery of the tarsal canal, which is a branch of the posterior tibial artery
c. Hawkin’s sign is a radiographic phenomenon seen 6-8 weeks post injury, and is associated with reduced incidence of avascular necrosis
d. The lateral process of the talus articulates with the posterior facet of the calcaneum
e. Valgus malunion is a frequently reported complication of these injuries

A

Answer: E

Talar neck fractures are high energy injuries to the hindfoot that are associated with a high incidence of talus avascular necrosis. Most common fracture of the talus (50%).

Foot is forced dorsiflexed with axial load.
Ipsilateral lower extremity fractures are common.

Inferior surface articulates with posterior facet of calcaneus.

Talar head articulates with navicular bone, sustenaculum tali, navicular bone, sustentaculum tali.

Lateral process articulates with posterior facet of calcaneus and lateral malleolus of fibula.

Posterior process consists of medial and lateral tubercles separated by groove for FHL.

Blood supply - talar neck receives blood supply via 3 sources:

  • Posterior tibial artery: via artery of tarsal canal (dominant supply - majority of talar body) and by the deltoid branch of PTA (supplies medial portion of talar body - and may be only remaining blood supply with a displaced fracture)
  • Anterior tibial artery: supplies head and neck
  • Perforating peroneal artery via artery of tarsal sinus: supplies head and neck

Hawkins Classification correlates degree of displacement to AVN risk.
I : Non-displaced 0-13%
II: Subtalar dislocation 20-50%
III: Subtalar and tibiotalar dislocation 20-100%
IV: Subtalar, tibiotalar & talonavicular dislocation (70-100%)

CT best Ix to determine degree of displacement, comminution and articular congruity. Will also show ipsilateral foot injuries (up to 89% incidence).

‘All’ cases require emergent closed reduction in ER.

ORIF: 2 approaches are recommended to visualise medial and lateral neck to assess reduction - typical areas of comminution are dorsal and medial
- Anteromedial: between tib an and posterior tibialis. Preserves soft tissue attachments, especially deep deltoid ligament (blood supply). Medial malleolar osteotomy also preserves deltoid ligament.
- Anterolateral: between tibia and fibula proximally, in line with 4th ray - elevate extensor digitorum breves and remove debris from subtalar joint.

Anatomic reduction is essential. Consider mini frag plate in comminuted fractures to buttress against varus collapse.

Complications:
- Osteonecrrosis: 31% overall (including all subtypes) - Hawkins sign is subchondral lucency best seen on mortise Xray at 6-8 weeks and indicates intact vascularity with resorption of subchondral bone - associated with talar neck comminution and open fractures
Delayed internal fixation is not associated with avascular necrosis

Post traumatic arthritis: subtalar (50%) and tibiotalar (33%)

Varus mal-union (25-30%) - can be prevented by anatomic reduction. Leads to decreased subtalar eversion (decreased motion with locked mid foot and hind foot). Weight bearing on the lateral border of the foot.
Treatment includes medial opening wedge osteotomy of talar neck.

78
Q
  1. Post-operative radiotherapy would be most appropriate for which of the following patients with Ewing’s sarcoma:

a. Intra-lesional resection margin in a patient without comorbidities
b. Excision of a tumour with a 95% tumour necrosis response to pre-operative chemotherapy with a 4mm excision margin
c. Pre-chemotherapy tumour volume excised
d. Excision of a tumour with an 85% tumour necrosis response to pre-operative chemotherapy with a 10mm excision margin
e. Patients receiving high dose busulphan-melphalan chemotherapy

A

Answer: D

Local recurrence significantly affects the overall survival in patients with a Ewing’s sarcoma. For those with a tumour in a limb, radiotherapy reduced the risk of local recurrence, especially in those with a marginal margin of excision, but the effect in central tumours was less clear. Radiotherapy for those who have had a wide margin of resection does not reduce the risk of local recurrence, regardless of the histological response to chemotherapy.

A Ewing’s sarcoma is a small round cell neuro-ectodermal neoplasm. It is the second most common primary bone tumour in children after osteosarcoma.1-3 The combination of neo-adjuvant chemotherapy and excision and/or radiotherapy for local primary control has become the mainstay of treatment for these tumours.4As a result of advances in surgical techniques, limb-salvage surgery is possible for most patients with a Ewing’s sarcoma following resection where appropriate.5,6 However, approximately 10% of patients suffer a local recurrence and the long-term survival for this group of patients remains less than 30%.7-10 There is evidence that surgical margins and the site of the tumour are significant prognostic factors for local recurrence, and the benefits of radiotherapy for reducing local recurrence have been described.11,12 However, the relationship between local recurrence and the response to chemotherapy has not been clearly documented and the role of adjuvant radiotherapy remain less clear.

In a recent large multicentre study, postoperative radiotherapy significantly reduced the rate of local recurrence compared with patients not receiving this treatment, and postoperative radiotherapy was recommended in patients with incomplete removal of the tissue as assessed by its volume before chemotherapy

79
Q
  1. One third of sarcomas are characterised by chromosomal translocations, resulting in the transcription of fusion genes and the production of fusion proteins. Detection of chromosomal translocations are useful for both diagnostic and prognostic purposes.

A tumour with a t11;22 translocation is most likely to be found in:

A. An elderly population
B. Long bones
C. Low grade tumours
D. Patients of Chinese descent
E. Patients that are female

A

Answer: B

80
Q
  1. The UK has been able to offer selected patient groups Proton Beam Therapy since December 2018.
    Which of the following patients would be most appropriate for Proton Beam Therapy?

a. Chordoma of the sacrum
b. Osteochondroma of the proximal tibia with malignant transformation
c. Osteiod osteoma of the femoral condyle
d. Osteosarcoma of the proximal femur
e. Synovial sarcoma affecting the foot

A

Answer: A

81
Q

A patient with Hereditary multiple osteochondromas discusses her risk of transmitting to her children during a consultation with you.
HMO is associated with:

a. De novo mutations in 50%
b. 25% chance offspring will have the disease
c. 50% chance offspring will have the disease
d. 50% of female offspring will have the disease
e. Only male offspring may be affected by the disease

A

Answer: C

82
Q

The use of Denosumab to augment treatment of giant cell tumours of the bone is increasing.
Which of the following statements regarding Denosumab would be most accurate?

a. Denosumab is an osteoprotegrin analogue, upregulating RANK-Ligand expression
b. Denosumab is a RANK-Ligand decoy receptor
c. Denosumab is a RANK=Ligand analogue
d. Denosumab is an osteoprotegrin monoclonal antibody
e. Denosumab is a RANK-Ligand monoclonal antibody

A

Answer: E

Monoclonal Ig2 against RANKL (inhibits binding of RANKL to RANK, like osteoprotegerin)

Bone metabolism is a dynamic process that balances bone formation and bone resorption.
- Central to this process is the RANK/RANKL/OPG pathway
Bone formation: stimulating osteoblasts and inhibiting osteoclasts

Bone resorption: performed by active osteoclast
- stimulated by RANKL in normal process
- stimulated by PTHrP in pathologic process (metastatic disease)

Osteoblasts produce:
- RANKL: which binds RANK and stimulates osteoclastic bone resorption.
- OPG: inhibits osteoclast differentiation, fusion and activation. Acts as a decoy receptor (is a competitive inhibitor of RANKL, and thus blocks it from activating osteoclasts)

Osteoclast inhibition decreases bone resorption.

Molecules that inhibit bone resorption:
- OPG
- Calcitonin : interacts directly with osteoclast via cell-surface receptors
- Estrogen (via decrease in RANK-L) - stimulates bone production (anabolic) and prevents resorption. Inhibits activation of adenylyl cyclase.
- TGF-beta (via increase in OPG)
- IL-10: suppresses osteoblasts

Osteoclast activation stimulates bone resorption.
Molecules that stimulate bone resorption:
- RANKL: secreted by osteoblasts and binds to the RANK receptor on osteoclast precursor and mature osteoclast cells.
- PTH: activation of its receptor stimulates adenylyl cyclase - binds to cell-surface receptors on osteoblasts to stimulate production of RANKL and M-CSF
- IL-1: stimulate osteoclast differention and thus bone resorption.
- 1,25 dihydroxy vit D: stimulates RANKL expression
- PGE2: activates adenylyl cyclase and stimulates resorption
- IL- 6 (myeloma)
- MIP-1A (myeloma)

Giant Cell Tumors are benign, aggressive tumors typically found in the epiphysis of long bones, most commonly at the distal femur and proximal tibia.
Patients typically present between ages 30 and 50 with insidious onset of pain of the involved extremity with activity, at night, or at rest.
Diagnosis is made with a biopsy showing mononuclear stromal cells that resemble interstitial fibroblasts with numerous giant cells dispersed throughout.

Metastases to lungs in 2-4% cases (wrist and hand lesions have greater chance of mets).

XR: eccentric lytic epiphyseal/metaphyseal lesion that often extends into the distal epiphysis and borders subchondral bone
“neo-cortex” is characteristic of benign aggressive lesions, and not unique to GCT

Medical management
- Bisphosphonates: osteclast inhibitors which may decrease the size of the defect in giant cell tumors and help prevent post-surgical recurrence

  • Denosumab: monoclonal antibody against RANK-ligand
    recent clinical trials suggest denosumab can decrease the size of the bone defect in giant cell tumor - 85-90% tumor necrosis
    Shows dramatic sclerosis and reconstitution of cortical bone after treatment.

Complications
bisphosphonates: esophagitis, gastritis, long term use can lead to atypical subtrochanteric femur fractures
denosumb: may cause nasopharyngitis, arthralgias. contraindications: severe hypocalcemia

83
Q
  1. A 27 year old decorator attends hospital after missing his footing on a ladder. This is his only injury. With respect to calcaneal fractures:

a. Conservation of the subtalar joint is the primary indication for surgery in this patient
b. An extended lateral approach provides the best surgical exposure to achieve the operative aim
c. Tongue-type variants can present greater clinical urgency than high energy, joint depression fractures
d. Smoking is not associated with skin problems where an extended lateral approach is not performed.
e. Once the calcaneus is fixed, the ankle should be splinted in a neutral position to prevent equinus contracture

A

Answer: C

84
Q
  1. Which of the following measurements is likely to be lower than normal in a varus knee with medial compartment osteoarthritis?

A. Mechanical lateral distal femoral angle (mLDFA)
B. Mechanical proximal tibial angle (mPTA)
C. Mechanical lateral distal tibial angle (mLDTA)
D. Anatomical lateral distal femoral angle (aLDFA)
E. Anatomical lateral distal tibial angle (aLDTA)

A

Answer: B

85
Q
  1. Which muscle is innervated by the anterior interosseous nerve?

a. Flexor carpi radialis
b. Pronator teres
c. Pronator quadratus
d. Supinator
e. Flexor carpi ulnaris

A

Answer: C

86
Q
  1. A hemi-hamate arthroplasty is a reconstructive procedure performed on which joint of the hand?

a. Carpometacarpal joint
b. Metacarpophalangeal joint
c. Proximal interphalangeal joint
d. Distal interphalangeal joint
e. Interphalangeal joint of thumb

A

Answer: C
Used in reconstruction of PIPJ pilon fractures

87
Q
  1. A 25 year old professional rugby player catching his ring finger on an opponents jersey sustaining an FDP avulsion injury.
    What type of injury is the most severe causing retraction of the FDP tendon into the palm?

a. Type 1
b. Type 2
c. Type 3
d. Type 4
e. Type 5

A

Answer: A

88
Q
  1. Which of the following fascial structures is NOT located in the finger?

a. Clelands ligament
b. Graysons ligament
c. Transverse fibres of Skoog
d. Oblique retinacular ligament of Landsmeer
e. Triangular ligament

A

Answer: C

Also known as transverse ligament of the palmar aponeurosis - thin band of transverse fibres of the distal portion of the palmar aponeurosis.

It runs deep and transverse to the longitudinally oriented pretendinous bands of the palmar fascial complex

oblique band (oblique retinacular ligament of Landsmeer)
function
links motion of DIP and PIP
lies volar to axis of PIP, but dorsal to axis of DIP
anatomy
origin: from lateral volar aspect of proximal phalanx,
insertion: to lateral terminal extensor dorsally (crosses collateral ligaments)
biomechanics
with PIP flexion, ligament relaxes to allow DIP flexion
with PIP extension, ligament tights to facilitate DIP extension
pathology
contracture causes volar displacement of lateral bands and a resulting Boutonniere Deformity
reconstruction of oblique retinacular ligament used to treat swan neck deformity
if ORL is tight,
resting finger position is DIP extended, PIP flexed
unable to flex DIP if PIP is extended
able to flex DIP only after PIP is flexed
contrast this with intrinsic tightness, where there is decreased PIP flexion when the MCP is extended, and improved PIP flexion when the MCP is flexed
contrast this with extrinsic tightnes (extensor tendon tightness), where there is increased PIP flexion when MCP is extended, and decreased PIP flexion when MCP is flexed

Digital cutaneous ligaments
Function
tether skin to deeper layers of fascia and bone to prevent excessive mobility of skin and improve grip
stabilize the digital neurovascular bundle with finger flexion and extension
Anatomic Components
Cleland’s ligaments (remember “C” for ceiling)
dorsal to digital nerves
not involved in Dupuytren’s disease
Grayson’s ligament (remember “G” for ground)
volar to digital nerves

Triangular ligament
Function
counteracts pull of oblique retinacular ligament, preventing lateral subluxation of the common extensor mechanism
Anatomy
triangular in shape
located on dorsal side of extensor mechanism, distal to PIP joint
Pathology
contracture leads to swan neck deformity

89
Q
  1. Which one of the following conditions is associated with the Delta phalanx?

a. Camptodactyly
b. Clinodactyly
c. Symphalagism
d. Syndactlyly
e. Polydactyly

A

Answer: B

90
Q
  1. A 18 year old netball player sustained an injury to her index finger PIPJ which the court side doctor reduces. She presents with the following finger deformity - clinical photograph of boutonierre deformity.
    What is her examination most likely to show?

a. Inability to extend the DIPJ
b. Fullness and triggering of the digit on flexion
c. PIPJ flexed, resistance to PIPJ extension causes DIPJ to become supple
d. PIPJ flexed, resistance to PIPJ extension causes DIPJ to become rigid
e. Volar subluxation of the middle phalanx and inability to extend the PIPJ

A

Answer: D

Central slip rupture – Elsons test

The common extensor divides at the PIP into a central slip and two lateral bands.

If the central slip is avulsed, the lateral bands will be freed, and may migrate volarly to actually become flexors of the finger. This leads to an odd appearance of the finger known as the Boutenniere deformity.

Elson’s test is the most well known test to diagnose an injury to the central slip before complications occur. Here’s how it’s done:

The patient bends the PIP 90° over the edge of a table and extends the middle phalanx against resistance.
In the absence of central slip injury (negative test), extension is strong AND the DIP remains floppy because the extension force is now placed entirely on maintaining extension of the PIP joint, so the lateral bands are not activated.
In the presence of central slip injury (positive test) there will be weak PIP extension, AND the DIP will extend abnormally and become rigid.

91
Q
  1. You are performing a total knee replacement for a patient with tricompartmental osteoarthritis. The angle indicated on the distal femoral cutting block is intended to create a cut perpendicular to what?

A. Femoral anatomical axis
B. Mechanical lateral distal femoral angle
C. Femoral mechanical axis
D. Femoral-tibial valgus angle
E. Joint line convergence angle

A

Answer: C

Q angle - line drawn from ASIS through middle of patella to tibial tuberosity.
Males: 13 deg
Females : 18 deg
In flexion: 8 deg

Sagittal ROM: 3 deg hyperextension to 155 deg flexion.
Normal gait requires ROM 0-70 deg

Instant centre of rotation = point at which the joint surfaces are in contact.
Posterior rollback - as the knee flexes, the instant centre of rotation on the femur moves posteriorly - allows for increased knee flexion by avoiding impingement.

Screw home mechanism: tibial ER 5deg in the last 15deg of extension. Occurs as medial tibial plateau articular surface is longer than lateral tibial plateau. This locks the knee, decreasing the work performed by the quadriceps while standing.

Normal anatomy:
Distal femur in ~9 degrees of valgus (anatomic axis compared to joint line)

5-7 deg valgus of femur refers to difference of anatomic axis to mechanical axis

proximal tibia is 2-3 degrees of varus (anatomic axis to joint line)

Technical goals
- restore mechanical alignment (mechanical alignment of 0°)
- restore joint line ( allows proper function of preserved ligaments. e.g., pcl)
- balanced ligaments (correct flexion and extension gaps)
- maintain normal Q angle (ensures proper patellar femoral tacking)

Mechanical axis of Limb = axis from centre of femoral head to centre of ankle

Anatomical axis femur: line that bisects medullary canal femur - determines entry point of femoral medullary guide rod.

Femoral Alignment:
Mechanical axis femur: line connecting centre of femoral head to point where anatomic axis meets intercondylar notch.
Obtaining neutral mechanical axis allows even load sharing between medial and lateral condyles of knee prosthesis.

Valgus cut angles: (~5-7° from AAF ) difference between AAF and MAF.
Perpendicular to mechanical axis - jig measures 6 deg from femoral guide (anatomic axis) - will vary if people very tall (<5deg) or very short (>7deg).

Tibial Alignment:
Anatomic axis tibia: line that bisects medullary canal, tibial medullary guide (internal or external) runs parallel to it. Determines entry point for tibial medullary guide rod.

Mechanical axis tibia: line from centre of proximal tibia to centre of talus. Proximal tibia is cut perpendicular to mechanical axis of tibia. Usually mechanical axis and anatomic axis of tibia are coincident, and therefore can usually cut the proximal tibia perpendicular to the anatomic axis (determined by intramedullary jig) - If there is tibia deformity and the mechanical and anatomic axis are not the same, then the proximal tibia must be cut perpendicular to the mechanical axis (i.e. use extramedullary tibial guide).

92
Q
  1. An 8 year old boy presents with a displaced comminuted unstable femoral shaft fracture. His weight is 42 kg.
    What treatment will most optimise the balance of risk and complications in this case?

a. Lateral entry rigid nail
b. Balanced traction
c. Titanium elastic nails (TENS)
d. Submuscular plating
e. External fixator

A

Answer: D

<6 months any fracture pattern -> pavlik harness (caution compression of femoral nerve if XS flexion - decreased quadriceps function) or early spica

Spica - flex hip 60-90deg and 30 abduction, ER needed to correct a rotational deformity.
Mould into recurvatum and valgus as muscular forces typically pull fracture into procurvatum and varus.
Goal of reduction: <10deg coronal plane and 20deg sagittal plane deformity, with no more than 2cm shortening or 10 deg rotational malalignment.

6 months - 5 years: traction +/- spica casting - relative contraindication in polytrauma, open fractures

TENS: 5-11 years, length stable, <49kgs.
Nail size by multiplying width of the isthmus femoral canal by 0.4 (80% canal fill). Entry site 2cm medially and laterally at level of distal physis, starting point 2cm proximal to physis.
Pain at insertion site nearest knee is commonest complication.

Submuscular plating: If length unstable fracture, >5years, >49kgs, very proximal or distal fractures, severe comminution.
Laterally based incision and plating with minimal disruption of soft tissue envelope. Small proximal and distal incisions - plate is placed between periosteum and vastus lateralis on the lateral side of the femur. Typically 12-16 hole 4.5mm narrow LC-DC plate, 3 screw proximal and 3 distal to the fracture - may need contouring.

Antegrade rigid intramedullary nail: >11 years, unstable fractures, >49kgs
Trochanter or lateral entry nail - avoid piriformis entry due to risk of injury to vascularity of femoral head (1-2% osteonecrosis)

External fixator: damage control orthopaedics in polytrauma, open fracture, associated vascular injuries requiring revascularization, segmental or significantly comminuted fractures.

Osteology
anterior bow to femur
isthmus is the narrowest portion of the femur

Muscles
iliopsoas creates a flexion and external rotation force on the proximal fragment
adductors create a shortening and varus force on the distal fragment

Biomechanics
femoral shaft cortical diameter and cortical thickness increase with age

93
Q

A 28 year old male fell off a ladder and presents with an isolated intracapsular proximal femur fracture centred on the base of the neck. He is active, a smoker and has no past medical history or other injuries.
What is the most appropriate management for this patient?

a. Non-operative
b. Total hip replacement
c. Internal fixation with cannulated screws
d. Internal fixation with DHS
e. Hemi-arthroplasty

A

Answer: D
DHS is biomechanically superior to cannulated screws when vertical fracture pattern in a young patient.
Has a lower re-operation rate vs cannulated screws in displaced femoral neck fractures, basic-cervical femoral neck fractures and current smokers.

Osteology
- normal neck shaft-angle 130 +/- 7 degrees
- normal anteversion 10 +/- 7 degrees

Blood supply to femoral head: major contributor is medial femoral circumflex (lateral epiphyseal artery) - some contribution to anterior and inferior head from lateral femoral circumflex. Also some contribution from inferior gluteal artery. Small and insignificant supply from artery of ligamentum teres

Displacement of femoral neck fracture will disrupt the blood supply and cause an intracapsular hematoma.

Pauwels Classification: (based on vertical orientation of fracture line)
Type I: <30 deg from horizontal
Type II: 30-50 deg from horizontal
Type III: >50deg from horizontal (most unstable with highest risk of non-union/AVN)

Cannulated screw fixation:
- Non displaced transcervical fractures
- Garden I or II in physiologically elderly
- Displaced transcervical fracture in younger patient (achieve reduction to limit vascular insult, must be anatomic so open if necessary)
- 3 screws - inverted triangle along calcar configuration shown to be superior to 2 screws - has a stronger fixation and a higher load to failure. Obtain as much screw spread as possible in femoral neck.

DHS:
- Basi-cervical fracture
- Vertical fracture pattern in a younger patient - biomechanically superior to cannulated screws. Consider use of anti-rotation screw. FAITH study.

Non-union 5-30% - increased incidence in displaced fractures. No correlation between age, gender and rate of non-union.
Varus malreduction most closely correlates with failure of fixation after reduction and cannulated screw fixation.
Tx: Valgus intertrochanteric osteotomy - turns vertical fracture line into horizontal fracture line, therefore decreases the shear forces across the fracture. Can be done even in presence of AVN as long as not several collapsed.

Free vascularised fibula graft - young patients with a viable femoral head.

Arthroplasty: older patients or when femoral head not viable.

94
Q
  1. Whilst performing the pubic osteotomy as part of periacetabular osteotomy, which structure is most likely to be injured?

a. Femoral nerve
b. Sciatic nerve
c. Obturator nerve
d. Lateral femoral cutaneous nerve
e. Pudendal nerve

A

Answer: C

The obturator nerve was likely to be injured during unprotected osteotomy of the pubis if the far cortex was penetrated by > 5 mm. This could be avoided by inclining the osteotome 45° medially and performing the osteotomy at least 2 cm medial to the iliopectineal eminence.

Nerve roots: L2-L4

Motor functions: Innervates the muscles of the medial compartment of the thigh (obturator externus, adductor longus, adductor brevis, adductor magnus and gracilis).

Sensory functions: Cutaneous branches of the obturator nerve innervate the skin of the medial thigh.

The obturator nerve is formed from the lumbar plexus. It receives fibres from the anterior divisions of L2, L3 and L4.

After its formation, the obturator nerve descends through the fibres of the psoas major and emerges from its medial border. It then travels posteriorly to the common iliac arteries and laterally along the pelvic wall – towards the obturator foramen of the pelvis.

The obturator nerve enters the medial thigh via the obturator canal (formed within the obturator foramen by the obturator membrane). It then divides into anterior and posterior branches:

Anterior division (anterior to the adductor brevis):
Descends in a plane between the adductor longus and adductor brevis (towards the femoral artery).
Here, it supplies motor fibres to the adductor longus, adductor brevis and gracilis. It can also supply the pectineus muscle.
It then pierces the fascia lata to become the cutaneous branch of the obturator nerve.

Posterior division (posterior to the adductor brevis):
Pierces the obturator externus muscle, and then descends in a plane between the adductor brevis and adductor magnus.
Innervates the obturator externus and adductor magnus muscles.

Reducing the risk of nerve injury during Bernese periacetabular osteotomy: a cadaveric study

The modified Smith-Petersen and Kocher-Langenbeck approaches were used to expose the lateral cutaneous nerve of the thigh and the femoral, obturator and sciatic nerves in order to study the risk of injury to these structures during the dissection, osteotomy, and acetabular reorientation stages of a Bernese peri-acetabular osteotomy. Injury of the lateral cutaneous nerve of thigh was less likely to occur if an osteotomy of the anterior superior iliac spine had been carried out before exposing the hip. The obturator nerve was likely to be injured during unprotected osteotomy of the pubis if the far cortex was penetrated by > 5 mm. This could be avoided by inclining the osteotome 45° medially and performing the osteotomy at least 2 cm medial to the iliopectineal eminence. The sciatic nerve could be injured during the first and last stages of the osteotomy if the osteotome perforated the lateral cortex of ischium and the ilio-ischial junction by > 10 mm. The femoral nerve could be stretched or entrapped during osteotomy of the pubis if there was significant rotational or linear displacement of the acetabulum. Anterior or medial displacement of < 2 cm and lateral tilt (retroversion) of < 30° were safe margins. The combination of retroversion and anterior displacement could increase tension on the nerve. Strict observation of anatomical details, proper handling of the osteotomes and careful manipulation of the acetabular fragment reduce the neurological complications of Bernese peri-acetabular osteotomy.

Malfunction of 1 of the 3 major nerves around the hip is a severe surgical complication of periacetabular osteotomy (PAO), and an understanding of the possible impacts of the different steps of the procedure on the nerves in their anatomic vicinity is essential to decrease the risk of these devastating injuries.

Step 1: Make a C-Shaped Skin Incision and Osteotomize the Anterior Superior Iliac Spine (ASIS) to Best Protect the LFCN

Step 2: Do Not Transect the Psoas Tendon to Enable an Easier Approach to the Ischium and Pubis as It Protects the Femoral Nerve from Being Overstretched. Sectioning the psoas tendon can result in potentially damaging tensioning of the femoral nerve.

Step 3: Prevent Injury to the Sciatic Nerve While the Osteotomy of the Ischium Is Being Performed from Anterior. We perform the first ischial cut without direct visual control; fluoroscopy can be used to visualize the position of the osteotomy, but it will not completely eliminate the risk of sciatic nerve injury.

Step 4: Protect the Obturator Nerve During Osteotomy of the Pubis

The obturator nerve and vessels cross through the obturator foramen inferior to the superior pubic ramus close to the level of the pubic osteotomy; as the nerve is fixed by a membrane in close vicinity to the undersurface of the pubic bone, it may become injured during pubic osteotomy.

Step 5: Prevent Injury to the Sciatic Nerve During Supra-Acetabular and Posterior Column (Inside-Out Osteotomy) for Complete Ischial Separation. When performing the supra-acetabular osteotomy, the abductors should be tunneled on the lateral aspect of the pelvis, retracted, and protected by a blunt Hohmann retractor.

Step 6: Prevent Femoral Nerve Injuries Resulting from Acetabular Reorientation

Most femoral nerve lesions are caused by kinking and overstretching during large acetabular fragment displacement.

Step 7: Avoid Sciatic Nerve Injury During Fragment Reorientation

Similar to the femoral nerve, the sciatic nerve can potentially be kinked and/or stretched during large displacement of the acetabular fragment; it can also be lanced by a bone spike on the fragment.

95
Q
  1. Which of the following best describes femoral component rotation when performing a mechanically aligned total knee replacement using measured resection?

a. Parallel to the posterior condylar axis
b. 3 degrees externally rotated to the trans-epicondylar axis
c. 3 degrees externally rotated to Whiteside’s line
d. 3 degrees externally rotated to the posterior condylar axis
e. Parallel to Whiteside’s line

A

Answer: D - the femoral prosthesis should be externally rotated 3 degrees from this axis to produce a rectangular flexion gap

Femoral prosthesis

There are three reference axis that one may use:

  • Anteroposterior axis: defined as a line running from the centre of the trochlear groove to the top of the intercondylar notch. A line perpendicular to this defines the neutral rotational axis
  • Transepicondylar axis: defined as a line running from the medial and lateral epicondyles - the epicondylar axis is parallel to the cut tibial surface. A posterior femoral cut parallel to the epicondylar axis will create the appropriate rectangular flexion gap
  • Posterior condylar axis: defined as a line running across the tips of the two posterior condyles. This line is in ~ 3 degrees of internal rotation from the transepicondylar axis, the femoral prosthesis should be externally rotated 3 degrees from this axis to produce a rectangular flexion gap. If the lateral femoral condyle is hypoplastic, use of the posterior condylar axis may lead to internal rotation of the femoral component
    WARNING: the average posterior condylar twist angle is 3º but the range is 1-10º. Therefore vary angle of femoral rotation based on variances in femoral anatomy.

Internal Rotation of Femoral Prosthesis will Increase Q angle: by internally rotating the femoral prosthesis, you are effectively bringing the groove and the patella medially. This will increase the Q angle to the tibial tubercle
will also make the medial compartment tight in flexion with subsequent TKA stiffness

Medialization of the Femoral Prosthesis will Increase Q angle: a medialized femoral prosthesis will bring the trochlear groove to a more medial position, and thus bring the patella medial with it, thus increasing the Q angle
therefore, you want the femoral component to be slighly lateral if anything

96
Q
  1. Which of the following should not be used to adress patella mal-tracking?

a. Increased tibial component external rotation
b. Lateral retinacular release
c. Increase femoral component external rotation
d. Downsize femoral component
e. Medialise patella button

A

Answer: D

As the Q angle increases from flexion to extension, the tension in the quadriceps and patellar tendons decreases. This occurs because the tibia rotates externally, thus moving the tibial tubercle laterally, in a mechanism known as screw-home. This relationship contributes to greater patellar instability in extension and in small degrees of flexion, causing patellar dislocation.

The vastus medialis oblique (VMO) muscle actively acts as a stabilizer, while the medial patellofemoral ligament (MPFL) and the lateral retinaculum are passive stabilizers. Imbalances may occur due to VMO muscle weakening or increased tension in the lateral retinaculum.
Although the lateral retinaculum contributes to only 10% of the lateral stability of the patella,12 when under excessive stress it often leads to abnormal contact of the lateral surface against the trochlea, increased lateral inclination of the patella (tilt), and pathological changes of the patellar tracking (maltracking).

Patellar prosthesis
The preferred position is either centered over the patella or medialized:
- medializing the patellar component is one strategy to decrease the Q angle. Results in uncoverage of lateral facet. Consider removing to lessen risk of lateral facet syndrome.
- Another alternative is use of an oval shaped patella with the apex medialized.

Lateralization of the patellar prosthesis will increase the Q angle and increase maltracking
Intraoperative lateral subluxation of the patella: if patella laterally subluxes intraoperatively during trialing, deflate tourniquet and recheck before performing a lateral release

Indications for resurfacing
absolute: inflammatory arthritis, patella maltracking, patellofemoral arthritis as the main indication for TKA

Internal rotation of the tibial component effectively results in relative external rotation of the tibial tubercle and an increase in the Q angle

it is critical to avoid techniques that lead to increase Q angle. Common errors include:
- Internal rotation of the femoral prosthesis
- Medialization of the femoral component
- Internal rotation or medialization of the tibial prosthesis
- Placing the patellar prosthesis lateral on the patella

97
Q
  1. Concerning the Mirel’s score for predicting pathological fractures, which of the following scenarios has the highest risk of fracture?

a. 55 year old man with prostate cancer and a 2mm isolated, non-tender lesion in the left greater trochanter which appears blastic on XR
b. Female patient with breast cancer and mild pain in both shoulders, XR shows lesions both humeri with mixed lytic and sclerotic features occupying just under 2/3 of the bone diameter
c. Elderly patient with lung cancer and a lytic lesion occupying 50% of the distal femur which is painful on weight bearing
d. Patient with myeloma, moderate pain and multiple tiny areas of bone loss throughout the right tibia and fibula
e. Asymptomatic sclerotic metastasis in the femoral shaft which occupy <10% of the whole bone diameter

A

Answer: C

Mirels’ criteria: 1//2//3
Site: upper limb // lower limb // peritrochanteric
Pain: mild // moderate // functional
Lesion: Blastic // mixed // lytic
Size: <1/3 // 1/3 to 2/3 // > 2/3

Score > 8 suggests prophylactic fixation.
do not proceed with fixation until primary neoplasm of bone has been ruled out with biopsy
goals of fixation
maximize ability for immediate mobilization and weight-bearing
protect the entire bone in setting of systemic or metastatic disease
optimize implant choice in the context of the patient’s overall prognosis.

following surgery refer the patient to radiation oncology for post-operative radiotherapy treatment to:
- decrease pain
- slow progression
- treat remaining tumor burden not removed at surgery

98
Q

A 64 year old man who is on enzalutamide for prostate cancer presents with lower back pain, he has the bone scan shown:
Choose the incorrect answer from options:

A. The figure shows a superscan appearance on a Tc-99 diphosphonate bone scan
B. The bone scan shows diffuse metastatic disease within the axial skeleton
C. Superscan appearance is a good prognostic sign
D. The appearance on the bone scan can be caused by Paget’s disease and lymphoma
E. The bone scan appearance is likely to account for his back pain.

A

Answer: C

A superscan is an imaging appearance on a Tc-99m diphosphonate bone scan which occurs as a result of a high ratio of bone to soft tissue tracer accumulation. Intense osteoblastic activity in the bones causes diminished renal and background soft tissue uptake.

This appearance can result from a range of causes:

diffuse metastatic disease
prostatic carcinoma
breast cancer
transitional cell carcinoma (TCC)
multiple myeloma (some difference in opinion)
lymphoma
patchy uptake nonetheless: look at skull and ribs
tends to somewhat spare the distal skeleton
metabolic bone diseases
renal osteodystrophy
hyperparathyroidism 1 (often secondary hyperparathyroidism)
osteomalacia
will involve distal skeleton
smoother uptake
myelofibrosis/myelosclerosis
mastocytosis
widespread Paget disease

Radiographic appearance
A metastatic superscan tends to have uptake throughout the axial skeleton and proximal appendicular skeleton, often somewhat heterogeneous. In contrast, a metabolic superscan tends to be more uniform and involve both the axial and more peripheral skeleton, including the distal extremities, calvarium, and mandible.

Little/no activity is seen within the urinary tract or soft tissues, as most of the tracer is concentrated within the bones.

99
Q
  1. Which of the following statements are correct concerning management of a solitary bone lesion in a patient with a history of malignancy?

A. Biopsy should be performed before definitive surgery is undertaken
B. Biopsy of a breast cancer metastasis with known bone involvement will never change management of the patient’s systemic cancer
C. Biopsy must be undertaken in sarcoma centres and there is no role for image-guided sampling
D. A ‘whoops’ procedure describes surgery for a bone lesion when the patient dies within 21 days of surgery
E. There is no role for MDT discussion as metastatic bone disease is a palliative condition

A

Answer: A

100
Q
  1. Which of the following is true regarding Ewing’s sarcoma?

a. The condition is usually resistant to radiotherapy
b. Less than 1% have metastases on presentation
c. Patients often present with pain and fever, and may be misdiagnosed with an infection
d. It is associated with a mutation in the EXT gene affecting chondrocytes of the growth plate
e. Bloods are usually normal

A

Answer: C

Ewing’s Sarcoma is a malignant, distinctive small round cell sarcoma associated with a t(11:22) translocation which most commonly occurs in the diaphysis of long bones in patients <25 with regional pain, swelling and fevers.
Diagnosis is made with a biopsy showing sheets of monotonous small round blue cells with prominent nuclei and minimal cytoplasm and immunostaining positive for CD99.
Treatment is usually neo-adjuvant chemotherapy and limb salvage surgical resection, followed by adjuvant chemotherapy +/- radiation.

t(11:22) translocation found in 85-95% of cases leads to the formation of a fusion protein (EWS-FLI1)

XR: large distractive lesion in diaphysis or metaphysics with ill-defined, permeative, moth-eaten appearance. Lytic lesion often with variable amounts of new bone formation.
Periosteal reaction may give onion skin or sunburst appearance. Large associated soft tissue mass in >80% cases.
Elevated ESR, WCC & LDH
Histology: monotonous small round blue cells, high nuclei: cytoplasm ratio. May have pseudo-rosettes (circles of cells with necrosis in centre).
Immunistaining: CD99 (+ve in 95%),

Standard of care:
- Chemo (neoadjuvant (8-12 weeks) + adjuvant (6-12 months))
-> surgery - goal is to obtain local control and prevent late recurrence of chemoresistant cells. Wide margins -> improved 5 year survival.
+/- adjuvant radioTx - not necessary if margins are adequate and there is a good response to chemo. Indicated if positive post- surgical resection margins or presence of pulmonary mets.

101
Q
  1. A 23-year-old male motorcyclist presents to the emergency department as a trauma call following a head on collision with a car. He has an isolated injury with a grossly deformed knee, with numbness in the dorsum of the foot and a weak posterior tibial pulse compared to the contralateral side. Following reduction and splinting of the knee, the pulse returns to normal. Which is the most useful investigation to assess associated injuries?

a. Plain radiographs of the knee
b. MRI of the knee
c. Plain CT of the knee
d. Ankle brachial pressure index
e. CT angio of the affected limb

A

Answer: E

Knee dislocations are high energy traumatic injuries characterized by a high rate of neurovascular injury.
Diagnosis is made clinically with careful assessment of limb neurovascular status. Radiographs should be obtained to document reduction.
Treatment is generally emergent reduction and stabilization with assessment of limb perfusion followed by delayed ligamentous reconstruction.

Associated injuries
- vascular injury
- nerve injury: usually common peroneal nerve injury (25% incidence), tibial nerve injury is less common
- fractures: present in 60% of dislocations
- soft tissue injuries: patellar tendon rupture, periarticular avulsion, displaced menisci

the knee is a ginglymoid joint and consists of tibiofemoral, patellofemoral and tibiofibular articulations

Ligaments: PCL, ACL, LCL, MCL, and PLC are all at risk for injury. Main stabilizers of the knee given the limited stability afforded by the bony articulation.

Blood supply: popliteal artery injuries occur often due to tethering at the popliteal fossa:
proximal - fibrous tunnel at the adductor hiatus
distal - fibrous tunnel at soleus muscle
N.B. geniculate arteries may provide collateral flow and palpable pulses masking a limb-threatening vascular injury

Biomechanics: the normal range of motion of 0-140 degrees with 8-12 degrees of rotation during flexion/extension

Kennedy classification based on the direction of displacement of the tibia:
Anterior (30-50%) = commonest. Hyperextension injury, usually involves PCL tear. Highest rate peroneal nerve injury, arterial injury generally intimal tear due to traction.

Posterior (30-40%): 2nd commonest, axial load to the flexed knee (dashboard injury), highest rate of vascular injury and highest incidence of complete tear of popliteal artery.

Lateral (13%) : varus or valgus force, usually tears of both ACL and PCL

Medial (3%): varus or valgus force, usually disrupted PLC & PCL

Rotational (4%): usually irreducible, posterolateral is commonest rotational dislocation, button holing (dimple sign) of femoral condyle through capsule. Is a contraindication to closed reduction due to risks of skin necrosis.

50% spontaneously reduced before arrival to ED.

Vascular exam priority - serial examinations, ABI.
If ABI >0.9 monitor with serial Ex
If ABI <0.9 perform arterial duplex USS or CT angiography

If pulses are absent or diminished:
- confirm that the knee joint is reduced or perform immediate reduction and reassessment
- immediate surgical exploration if pulses are still absent following reduction: ischemia time >8 hours has amputation rates as high as 86%
imaging contraindicated if it will delay surgical revascularization

102
Q
  1. What surgical procedure would you offer to an 18 year old female with hip pain and disability? Plain XR shows a lateral central edge angle of 18 degrees and an acetabular index of 20 degrees. Hip joint space is well maintained. Femoral anteversion is 15 degrees.

a. Salter osteotomy
b. Femoral osteotomy
c. Chiari osteotomy
d. Peri-acetabular osteotomy
e. Iliac osteotomy

A

Answer: D

hip dysplasia
acetabular index (AI) : angle formed by Hilgenreiner’s line (horizontal line through the right and left triradiate cartilage) and a line from a point on the lateral triradiate cartilage to a point on lateral margin of acetabulum - should be < 25° in patients older than 6 months

center-edge angle (CEA) of Wiberg : assesses superolateral coverage of the femoral head on the AP view.
angle between a verticle line through the center of the femoral head and the acetabular edge : < 20° is considered abnormal
only reliable in patients > 5 years old

periacetabular osteotomy (PAO) for symptomatic dysplasia in an adolescent or adult with a concentrically reduced hip and congruous joint space with a preserved range of motion. Triradiate cartilage must be closed.
Intraoperative dynamic testing of hip motion is needed to determine the need for femoral osteotomy (minimum of 90° flexion and 15° internal rotation to prevent FAI)
Multiple osteomies in pubis, ilium and ischium near the acetabulum, technically most challenging.
Advantages:
provides hyaline cartilage coverage
preserved integrity of the posterior column, which allows patients to weight bear as tolerated postoperatively (posterior column and pelvic ring remain intact).
large multidirectional corrections
preserves external rotators
delays need for arthroplasty

outcomes
reliably improves radiographic parameters and symptomatology
92% survivorship at 15 years in avoiding THA

Chiari = salvage osteotomy for unreduced hip - recommended for patients with inadequate femoral head coverage and incongruous joint (concentric reduction cannot be obtained) - Osteotomy starts above the acetabulum to the sciatic notch and ileum is shifted lateral beyond the edge of the acetabulum.
Depends on fibrocartilage
metaplasia for successful results.
Medializes the acetabulum via iliac osteotomy

Shelf: salvage in patients >8 years, add bone to the lateral weight bearing aspect of the acetabulum by placing extra-articular buttress of bone over the subluxed femoral head.
Depends of fibrocartilage metaplasia for successful results.

Salter - redirectional: younger patient with open triradiate cartilage - single transverse cut above the acetabulum through the ilium to sciatic notch. Acetabulum then hinges through pubis symphysis - improves anterolateral coverage (may length leg up to 1cm)
Triple (Steele) is Salter with additional cuts through superior and inferior rami - redirectional for anterolateral coverage)

Dega - volume reducing. favoured in neuromuscular dislocation and patients with posterior acetabular deficiency - for severe cases.
Osteotomy from acetabular roof to triradiate cartilage (incomplete cuts through peri-capsular of the innominate bone).
Acetabulum hinges through the triradiate cartilage. Does not enter the sciatic notch and is therefore stable and does not require internal fixation.
Improves anterior, central or posterior coverage.

Pemberton = volume reducing, moderate to severe. Triradiate cartilage must be open. stable as osteotomy does not enter sciatic notch.

103
Q
  1. This 20 year old man is injured in a motor vehicle accident and presents with an isolated injury to his knee. CT and MRI scan are done - PCL Avulsion injury. He has a Grade III posterior draw test.

a. Open posteromedial approach and fixation with screw and washer +/- suture augmentation
b. Knee arthroscopy and trans-tibial tunnel suture fixation
c. Anterolateral approach and fixation with screw and washer +/- suture augmentation
d. Brace and physiotherapy
e. Cast immobilisation 6 weeks

A

Answer: A

Conservative management for PCL avulsion -> early degenerative changes, meniscal tears and chondral damage. Poor functional outcomes.

The open posteromedial approach, which avoids nerves and blood vessels, has the potential to provide a clearer view of the broken ends of the fractured bone, and allows for complete fracture reduction and successful fixation of the bone fracture fragments. Arthroscopic reduction is difficult to achieve.

PCL injuries are traumatic knee injuries that may lead to posterior knee instability and often present in combination with other ipsilateral ligamentous knee injuries.

PCL is the primary restraint to posterior tibial translation and functions to prevent hyperflexion/sliding. Isolated injuries cause the greatest instability at 90° of flexion.

Associated conditions
- combined PCL and posterolateral corner (PLC) injuries
- multiligamentous knee injuries
- knee dislocation

PCL origin: posterior tibial sulcus below the articular surface
insertion: anterolateral medial femoral condyle
broad, crescent-shaped footprint
38 mm in length x 13 mm in diameter - PCL is 30% larger than the ACL

PCL has two bundles:
- Anterolateral bundle: tight in flexion. Strongest and most important for posterior stability at 90° of flexion
mnemonic “PAL” - PCL has an AnteroLateral bundle (N.B. ACL is AM & PL bundle)
- Posteromedial bundle: tight in extension - reciprocal function to the anterolateral bundle
PCL lies between the meniscofemoral ligaments: ligament of Humphrey (anterior) and ligament of Wrisberg (posterior) - originate from the posterior horn of the lateral meniscus and insert into PCL substance

Blood supply by branches of the middle geniculate artery and fat pad

PCL injury Classification:(based on posterior subluxation of tibia relative to femoral condyles with knee in 90° of flexion)
Grade I: partial tear, 1-5mm posterior tibial translation, tibia remains anterior to the femoral condyles.
Grade II: complete isolate tear, 6-10mm posterior tibial translation. Complete injury in which the anterior tibia is flush with the femoral condyles.
Grade III: combined PCL & capsuloligamentous injury, Ex >10mm posterior tibial translation. Tibia is posterior to the femoral condyles and often indicates an associated ACL and or PLC injury.

PCL repair of bony avulsion fractures or reconstruction

104
Q

A 50 year old lady underwent fixation of a distal femur fracture with a lateral locking plate.
She developed pain and hardware irritation due to prominent medial screw ends.
Which of the following intra-operative fluoroscopy imaging would have prevented this complication?

a. AP views with 45 degrees knee flexion
b. AP views with 30 degrees internal rotation
c. True Lateral views in flexion
d. AP views in full extension
e. AP views with 30 degrees external rotation

A

Answer: B

105
Q

A 43 year old patient presents with an open tibial fracture following a road traffic accident. On examination a 6cm wound with a moderate amount of grit contamination is noted over the fracture site and pedal pulses are palpable. Which of the following statements is true in relation to the injury?

a. The patient does not have compartment syndrome as she has pedal pulses
b. Even when wound closure or coverage cannot be achieved at the time of first surgery, definitive fixation is indicated if possible to stabilise the bone ends and reduce infection risks, with VAC dressing coverage pending definitive soft tissue coverage
c. Debridement should be performed within 6 hours of injury as otherwise the risk of infection increases significantly
d. Compartment syndrome is unlikely to occur with an open injury since the compartments are decompressed through the wound
e. Prior to formal debridement, the wound should only be handled to remove gross contamination and dressed with saline soaked gauze. Mini-washouts in the emergency department are not indicated

A

Answer: E

106
Q

A 32-year-old carpenter sustains a penetrating drill bit injury to the volar surface of the wrist at the level of the carpal tunnel, with a complete transection of the traversing nerve in the carpal tunnel but no tendon injuries and no injuries to superficial structures on exploration. The nerve is repaired. What are the expected examination findings in the immediate post-operative period? (I.e., prior to any healing)

a. Sensory deficit to the radial 3 and a half digits and thenar eminence. Loss of thumb opposition and resting posture with extension at the index and middle finger MCPJs
b. Sensory deficit to the radial 3 and a half digits and thenar eminence. Preserved thumb opposition and resting posture with extension at the index and middle finger MCPJs
c. Sensory deficit to the radial 3 and a half digits, sparing the thenar eminence. Preserved thumb opposition and resting posture with extension at the index and middle finger MCPJs
d. Sensory deficit to the radial 3 and a half digits, sparing the thenar eminence. Loss of thumb opposition and resting posture with extension at the index and middle finger MCPJs
e. Sensory deficit to the radial 3 and a half digits, sparing the thenar eminence. Loss of thumb opposition and resting posture with extension at the index and middle finger PIPJs

A

Answer: D

107
Q

Concerning the management of patients with pelvic ring fractures:

A. Reconstruction of the pelvic ring should occur within 24 hours of stabilisation of the patient’s physiological state
B. Patients who are admitted that require surgical stabilisation should transferred to a specialist centre within 4 hours
C. Post binder xray views are not required after a negative CT scan
D. Binder removal should occur within 24 hours of injury
E. Blood stained urine mandates a bladder ultrasound

A

Answer: D

108
Q
  1. Which structure is most commonly at risk whilst creating an anterolateral portal for ankle arthroscopy?

a. Dorsalis pedis artery
b. Superficial peroneal nerve
c. Tibialis anterior tendon
d. Extensor digitorum tendon
e. Peroneus tertius tendon

A

Answer: B

Portals:
- Anteromedial: primary viewing portal (typically established first) and gives access to anteromedial joint.
 Medial to tibialis anterior
 Lateral to medial malleolus
 Make portal between tibialis anterior and saphenous vein

• Anterolateral: gives access to anterolateral joint
 Just lateral to peroneus tertius and superficial peroneal nerve
 Medial to lateral malleolus
 can trace out superficial peroneal nerve prior to incision

109
Q
  1. A 22 year old female with hip pain is found to have an excessive internal rotation of 80 degrees and external rotation of 10 degrees She has been thoroughly investigated with CT and MRI and has exhausted all non-operative measures. What is the most appropriate surgical treatment option?

a. Hip arthroscopy and labral repair
b. Valgus femoral osteotomy
c. Derotating femoral osteotomy
d. Varus femoral osteotomy
e. Peri-acetabular osteotomy

A

Answer: C

Femoral Anteversion is a common congenital condition caused by intrauterine positioning which lead to increased anteversion of the femoral neck relative to the femur with compensatory internal rotation of the femur.
Diagnosis is made clinically with the presence of intoeing combined with an increase in internal rotation of the hip of greater than 70° (normal is 20-60°) with an accompanying decrease in external rotation of the hip of less than 20° (normal is 30-60°).

Femoral anteversion is characterized by
increased anteversion of the femoral neck relative to the femur
compensatory internal rotation of the femur
lower extremity intoeing

In younger children : observation and parental reassurance - most cases spontaneously resolve by age 10.

Operative Mx with femoral de-rotation osteotomy - for >§0years with <10 degrees eexternal rotation, typically performed at the intertroch level

110
Q
  1. A patient has open reduction and internal fixation of a displaced radial neck fracture with a plate and screws. To avoid impingement on the proximal ulna, what surface should the plate be located?

a. Within a 90 degree arc or safe zone
b. Within a 120 degree arc or safe zone
c. Within a 180 degree arc or safe zone
d. 1cm distal to radial head articular surface
e. 2cm distal to the radial head articular surface

A

Answer: A

Posterolateral plate placement:
- Safe zone (non-articular area) consists of 90-110º arc defined by the projections of the radial styloid and Lister’s tubercle

This zone is straight lateral with arm in neutral rotation to avoid impingement of ulna with forearm rotation

Bicipital tuberosity is the distal limit of plate placement - anything distal to that will endanger PIN

Countersink implants on articular surface
Plate removal relatively common in order to restore forearm rotation

Complications
- PIN injury
- destabilization of
-lateral ligament complex
- articular surface penetration with screws
- mechanical block to motion by hardware

111
Q
  1. A 19 year old female presents to the elective clinic with intermittent generalised left knee pain, and a large recurrent effusion, with no history of injury and no mechanical symptoms. There is no structural deficit identified through clinical examination, but MRI does demonstrate an abnormal synovial appearance.
    What is the most likely diagnosis?

a. Discoid meniscus
b. Synovial chondromatosis
c. Meniscal root tear
d. PVNS
e. Juvenile idiopathic arthritis

A

Answer: D

Pigmented Villonodular Synovitis is a locally aggressive neoplastic synovial disease (not a true neoplasm) characterized by joint effusions, expansion of the synovium, and bony erosions.
The condition usually presents in patients between 30 and 40 years old with recurrent atraumatic knee hemarthrosis.
Diagnosis is multifaceted with clinical assessment for joint effusion (most commonly the knee), MRI studies showing synovial expansion, arthrocentesis revealing a brown fluid, and biopsy revealing hemosiderin-stained multinucleated giant cells.

Treatment generally consists of partial or total surgical synovectomy depending on presence of localized or diffuse PVNS. PVNS is associated with a high rate of recurrence and accelerated degenerative changes of the knee ultimately requiring arthroplasty
TKA in patients with PVNS is associated with complication rates

Pathobiology: caused by an overexpression of CSF1 gene. Overexpression leads to clusters of aberrant cells creating focal areas of soft tissue hyperplasia in the synovial cells lining joints.
A locally aggressive neoplastic synovial disease (not a true neoplasm).
Mutation Chr 1p13 in majority cases.

Recurrence is the most frequent complication for both intra-articular and extra-articular disease: 30%-50% recurrence rate despite complete synovectomy - same rates for complete open vs open+arthroscopic.

Synovial Chondromatosis is a proliferative disease of the synovium associated with cartilage metaplasia that results in multiple intra-articular loose bodies.
The condition usually presents in patients between 30 and 50 years of age with localized joint pain, stiffness, and swelling.
Diagnosis is made on radiographs in late disease but MRI studies may be required in early disease to show cartilage nodules throughout the joint space.

112
Q
  1. Which of the following has been demonstrated to be significantly better at 5 years when comparing partial knee replacement with total knee replacement for medial knee arthritis?

A. Cost effectiveness
B. Oxford knee score
C. Patient satisfaction
D. Revision rate
E. Operation duration

A

Answer: A

113
Q
  1. A 22-year-old male sustains a non-contact twisting injury to his knee playing football, within 30 minutes he has an effusion and has been unable to continue playing. What is the most likely associated injury to occur at the same time as the primary structure injured?

a. Medial meniscus tear
b. Posterior cruciate ligament tear
c. Anterior cruciate ligament tear
d. Lateral meniscus tear
e. Lateral collateral ligament tear

A

Answer: A

114
Q
  1. A 12-month-ol child becomes acutely unwell with a painful knee following varicella infection. What organism must you rule out?

a. Group A beta-haemolytic streptococcus
b. Group B beta-haemolytic streptococcus
c. Group C beta-haemolytic streptococcus
d. Group G beta-haemolytic streptococcus
e. Group D beta-haemolytic streptococcus

A

Answer: A

The most common bacterial etiologic agent following varicella is group A beta-hemolytic streptococcus. The other organisms are much less common. Staphylococcus aureus is the most common bone infection organism. Staphylococcus epidermidis is increasingly a bone infection organism. Group B streptococcus occurs more commonly in newborns. Kingella kingae is a common joint pathogen but is not as common following varicella.

a subperiosteal abscess develops when the purulence breaks through the metaphyseal cortex
septic arthritis develops when the purulence breaks through an intra-articular metaphyseal cortex (hip, shoulder, elbow, and ankle) (NOT KNEE)

Staph aureus is the most common organism in all children
strains of community-acquired (CA) MRSA have genes encoding for Panton-Valentine leukocidin (PVL) cytotoxin
PVL-positive strains are more associated with complex infections, multifocal infections, prolonged fever, abscess, DVT, and sepsis
MRSA is associated with increased risk of DVT and septic emboli

Group B Strep: is most common organism in neonates

Kingella kingae: becoming more common in younger age groups

Pseudomonas: is associated with direct puncture wounds to the foot

H. influenza: has become much less common with the advent of the Haemophilus influenza vaccine

Mycobacteria tuberculosis: children are more likely to have extrapulmonary involvement
biopsy with stains and culture for acid-fast bacilli is diagnostic

Salmonella: more common in sickle cell patients

115
Q
  1. You review a non-weight bearing child with a temperature of 38 degrees and an ESR of 57mm/hr. What is the probability they have septic arthritis?

a. 3%
b. 40%
c. 80%
d. 93%
e. 99%

A

Answer: B
Kochers criteria

Original Kocher Criteria
four criteria
WBC > 12,000 cells/µl of serum
inability to bear weight
fever > 101.3° F (38.5° C)
ESR > 40 mm/h

algorithm
probability of septic arthritis may be as high as 99.6% when all four criteria above are present
if none of the above predictors are present, probability of having septic arthritis is <0.2%
3% incidence of septic arthritis if 1/4 criteria present, 40% incidence if 2/4 criteria present, 93% incidence if 3/4 criteria present

Modified Kocher Criteria
additional criteria
CRP

116
Q
  1. You see a 15 year old elite female athlete in your clinic following netball injury. She has an MRI scan which confirms your suspicion. She has had intense physiotherapy and is keen to proceed with operative intervention.
    What procedure would you offer her?

a. Arthroscopic trimming of the meniscus
b. ACL reconstruction
c. Repair of torn meniscus
d. PCL and PLC reconstruction
e. Autograft OATS procedure

A

Answer: B

ACL -> anterior and lateral rotatory instability of the knee.

Females tend to sustain ACL injuries at a younger age than males, and more frequently on the supporting leg (vs males on kicking leg). Preponderance for females due to landing biomechanics and neuromuscular activation patterns (quadriceps dominant) play the biggest role.

  • Valgus moment at knee and adduction moment at hip upon landing.

Pathoanatomy:
- Non-contact pivoting injury - tibia translates anteriorly while knee is in slight flexion and valgus.
- Blow to the lateral aspect of the knee

Associated injuries:
- Lateral meniscal tears in 54% of acute ACL tears, medial in chronic causes.
- PCL, LCL/PLC injuries
- Chronic ACL deficient knees associated with chondral injuries and complex unrepairable meniscal tears and bucket handle medial meniscus tears.

2 bundles: combined 32mm length x 7-12 mm width.

Bundles named for tibial attachment.
- Anteromedial bundle: more isometric, tightest in flexion, primarily responsible for restraining anterior tibial translation (anterior drawer test)
- Posterolateral bundle: greater length changes, tightest in extension, primarily responsible for rotational stability (pivot shift test)

Femoral attachment
- Lateral intercondylar ridge demarcates the anterior edge of the ACL
- Bifurcate ridge separates the anteromedial and posterolateral bundle attachment

Tibial attachment: anterior tibia, between intercondylar eminences.

90% Type I collagen, 10% type II collagen.
Blood supply = middle geniculate artery
Innervation: Posterior articular nerve (branch of tibial nerve)

Provides 85% stability to prevent anterior translation of the tibia relative to the femur and acts as a secondary restraint to tibial rotation and varus/valgus rotation. 2200N strength (anterior).

ACL rupture: pop, deep knee pain, immediate swelling (70%), haemarthrosis.
Quadriceps avoidance gait (does not actively extend knee).
Coronal or sagittal plane deformity - varus deformity increases risk for SCL re-rupture..

Lacman’s test: most sensitive exam test - Grading A= firm end point, B = no end point.
1: 3-5mm translation
2 A/B: 5-10mm translation
3 A/B: > 10mm translation
PCL tear may give a false positive Lacman due to posterior subluxation.

Pivot Shift: knee brought from extension (anteriorly subluxated) to flexion (reduced) with valgus and internal rotation of the tibia - reduces at 20-30 deg flexion due it IT band tension - must be completely relaxed patient (/anaesthetised) and mimics actual giving way event.

Segond fracture: avulsion fracture of the proximal lateral tibia - pathognomonic for ACL tear
Deep Sulcus (terminalis) sign: depression on the lateral femoral condyle at the terminal sulcus, a junction between the weight bearing tibial articular surface and the patellar articular surface of the femoral condyle.

Treatment: physio, lifestyle modifications in low demand patients and recreational athletes not participating in cutting/pivoting activities, BUT linked to increased meniscal & cartilage damage - loss of meniscal integrity, more frequent buckling episodes.

ACL reconstruction: must have full ROM restored following injury (unless meniscal tear causing mechanical block) - lack of pre-op ROM risk factor for post-op arthrofibrosis.
Indicated in younger more active patients, children, older active patients, partial/single bundle tears with clinical and functional instability.

Goal: anatomically reconstruct ligament to restore anterior and rotational stability.
Femoral tunnel:
- Sagittal plane: 1-2mm rim of bone between the tunnel and posterior cortex of the femur.
- Coronal plane: lateral wall at 2 o’clock (L) or 10 o’clock (R) - creates a more horizontal graft (and reduces rotational laxity).

Tibial Tunnel:
- Sagittal plane: centre of tunnel entrance into joint should be 10-11mm in front of the anterior border of PCL insertion, 6mm anterior to the median emminence, 9mm posterior to the inter meniscal ligament.
- Coronal plane: tunnel trajectory of <75deg from horizontal (obtain by moving tibial starting point halfway between tibial tubercle and posterior medial edge of tibia.)

Graft failure rate approx 5% - tunnel malpoisiton is commonest cause (70%).

Graft pre-conditioning can reduce stress relaxation up to 50%.
Quadrupled hamstring autograft: small incision, less peri-op and anterior knee pain. Max load to failure 4000N. Decreased peak flexion strength at 3 years compared to BPTB - concern about hamstring weakness in female athletes leading to increased risk of re-rupture. Risk saphenous nerve branches during harvest.

Concurrent pathology:
- MCL: if low grade MCL injury amenable to non-operative treatment, allow MCL to heal prior to ACL reconstruction. If high grade MCL injury necessitating repair/reconstruction, may be done concurrently with ACL. Failure to address valgus instability can jeopardise ACL graft with higher re-rupture rates.

  • Meniscal tears: perform repair or meniscectomy at time of ACL recon - increased meniscal healing rate when repaired at the same time as ACL.
  • Chondral injuries:
    Presence of chondral defects consistently lowers long term PROMs following ACL recon.
  • PCL and PLC injuries: concurrent reconstruction or staged procedure - failure to address will lead to varus instability and ACL graft overload.
117
Q
  1. A 5 year old boy presents with a 4 day history of sore throat and cough. Over the last 24 hours, the parents have noticed that he has a ‘wry neck.’ Plain XR and subsequent CT demonstrate atlanto-axial rotatory displacement.
    What is the most appropriate treatment option?

a. Excision of tumour
b. Immobilisation and NSAIDs
c. Stretching exercises
d. Posterior fusion
e. Observation

A

Answer: B

118
Q
  1. With regard to Adolescent Idiopathic Scoliosis, which curve is most common?

a. Right thoracic
b. Right thoracic and left lumbar
c. Left lumbar
d. Left thoracic
e. Long C-shaped curve

A

Answer: A

119
Q

What is the most appropriate treatment in a child with SUFE with a 2 week history of pain, able to walk with crutches, and with a slip angle of 50 degrees:

a. Pinning in situ with a single cannulated screw on a planned list
b. Pinning with a cannulated screw following closed reduction
c. Osteotomy on a semi-urgent elective list
d. Immediate pinning in situ on an emergency list
e. Open reduction and pinning

A

Answer: A

120
Q

In DDH, what is the name of the horizontal line drawn through the tri-radiate cartilage on a pelvic radiograph?

a. Shenton
b. Perkin
c. Acetabular index
d. Hilgenreiner
e. Klein

A

Answer: D

121
Q

Which of the following pelvic osteotomies can be said to be volume reducing?

A. Salter Innominate
B. Pemberton osteotomy
C. Staheli shelf osteotomy
D. Ganz
E. Steel osteotomy

A

Answer: B

Pemberton = volume reducing, moderate to severe. Triradiate cartilage must be open. stable as osteotomy does not enter sciatic notch.
Osteotomy starts approximately 10-15mm above the AIIS and proceeds posteriorly to end at the level of the ilioischial limb of the triradiate cartilage (halfway between the sciatic notch and the posterior acetabular rim).
Acetabulum hinges at the triradiate cartilage posteriorly and the symphysis pubis anteriorly.
Does not enter the sciatic notch and is therefore stable and does not need internal fixation.
Improves anterolateral coverage.

periacetabular osteotomy Ganz (PAO) for symptomatic dysplasia in an adolescent or adult with a concentrically reduced hip and congruous joint space with a preserved range of motion. Triradiate cartilage must be closed.
Intraoperative dynamic testing of hip motion is needed to determine the need for femoral osteotomy (minimum of 90° flexion and 15° internal rotation to prevent FAI)
Multiple osteomies in pubis, ilium and ischium near the acetabulum, technically most challenging.
Advantages:
provides hyaline cartilage coverage
preserved integrity of the posterior column, which allows patients to weight bear as tolerated postoperatively (posterior column and pelvic ring remain intact).
large multidirectional corrections
preserves external rotators
delays need for arthroplasty

outcomes
reliably improves radiographic parameters and symptomatology
92% survivorship at 15 years in avoiding THA

Chiari = salvage osteotomy for unreduced hip - recommended for patients with inadequate femoral head coverage and incongruous joint (concentric reduction cannot be obtained) - Osteotomy starts above the acetabulum to the sciatic notch and ileum is shifted lateral beyond the edge of the acetabulum.
Depends on fibrocartilage
metaplasia for successful results.
Medializes the acetabulum via iliac osteotomy

Shelf: salvage in patients >8 years, add bone to the lateral weight bearing aspect of the acetabulum by placing extra-articular buttress of bone over the subluxed femoral head.
Depends of fibrocartilage metaplasia for successful results.

Salter - redirectional: younger patient with open triradiate cartilage - single transverse cut above the acetabulum through the ilium to sciatic notch. Acetabulum then hinges through pubis symphysis - improves anterolateral coverage (may length leg up to 1cm)

Triple (Steele) is Salter with additional cuts through superior and inferior rami - redirectional for anterolateral coverage)

Dega - volume reducing. favoured in neuromuscular dislocation and patients with posterior acetabular deficiency - for severe cases.
Osteotomy from acetabular roof to triradiate cartilage (incomplete cuts through peri-capsular of the innominate bone).
Acetabulum hinges through the triradiate cartilage. Does not enter the sciatic notch and is therefore stable and does not require internal fixation.
Improves anterior, central or posterior coverage.

hip dysplasia
acetabular index (AI) : angle formed by Hilgenreiner’s line (horizontal line through the right and left triradiate cartilage) and a line from a point on the lateral triradiate cartilage to a point on lateral margin of acetabulum - should be < 25° in patients older than 6 months

center-edge angle (CEA) of Wiberg : assesses superolateral coverage of the femoral head on the AP view.
angle between a verticle line through the center of the femoral head and the acetabular edge : < 20° is considered abnormal
only reliable in patients > 5 years old

122
Q

Which of the following statements about tarsal coalition is true?

a. Talocalcaneal coalition is the commonest form of tarsal coalition
b. The management of choice is surgical excision
c. Cross sectional imaging should always be performed prior to considering surgery
d. Orthotics should be prescribed
e. A coalition is almost always symptomatic

A

Answer: C

123
Q

A 13 year old girl presents with progressive development of cavus feet.
What would be the most appropriate investigation when she is first seen by you in clinic?

a. Nerve conduction velocity studies
b. Biopsy of the quadriceps femoris muscle
c. Biopsy of the sural nerve
d. DNA testing
e. Chromosomal analysis

A

Answer: A

124
Q

Regarding the Ponseti method, the stages in correction of the clubfoot deformity proceed in the following order:

A. Elevation of the first ray to correct cavus, abduction, then dorsi-flexion with a Tendo-achilles tenotomy
B. Pronation, dorsiflexion, abduction, Tendo-achilles tenotomy
C. Correction of varus, then supination then dorsiflexion with a Tendoachilles tenotomy
D. Elevation of the first ray to correct cavus, then dorsiflexion with a TA tenotomy followed by a final abduction cast
E. Elevation of the first ray to correct cavus, then dorsiflexion with a TA tenotomy

A

Answer: A
CAVE

Clubfoot, also known as congenital talipes equinovarus, is a common idiopathic deformity of the foot that presents in neonates.
Diagnosis is made clinically with a resting equinovarus deformity of the foot.
Half of cases are bilateral and in 80%, clubfoot is an isolated deformity.
Genetic component is strongly suggested - recent link to PITX1.
Associated with arthrogryposis.

Pathophysiology
Muscle contractures contribute to the characteristic deformity that includes (CAVE):
- Cavus (tight intrinsics, FHL, FDL)
- Adductus of forefoot (tight tibialis posterior)
- Varus (tight tendoachilles, tibialis posterior, tibialis anterior)
- Equinus (tight tendoachilles)

Bony deformity consists of medial spin of the midfoot and forefoot relative to the hindfoot
- talar neck is medially and plantarly deviated
- calcaneus is in varus and rotated medially around talus
- navicular and cuboid are displaced medially

125
Q
  1. When suspecting non-accidental injury, when should a skeletal survey be performed and reported?

a. Within 7 days of requesting
b. Within 72 hours of requesting
c. Within 24 hours of requesting
d. Within 48 hours of requesting
e. Within 12 hours of requesting

A

Answer: C

126
Q

In early Perthe’s disease, which of the following features would you expect to see on pelvic XRs?

a. Bilateral symmetrical changes
b. Early acetabular changes
c. Irregular delayed proximal femoral ossification centres
d. Smaller sclerotic epiphysis with medial joint space widening
e. Femoral head deformity with widening and flattening

A

Answer: D